Số nguyên tố và các bài toán liên quan phương pháp giải bài toán số học trong các kì thi hsg

Page 1

SỐ NGUYÊN TỐ VÀ CÁC BÀI TOÁN LIÊN QUAN

vectorstock.com/24597468

Ths Nguyễn Thanh Tú eBook Collection

Số nguyên tố và các bài toán liên quan phương pháp giải bài toán số học trong các kì thi học sinh giỏi toán quốc tế, quốc gia Việt Nam và các nước trên thế giới WORD VERSION | 2022 EDITION ORDER NOW / CHUYỂN GIAO QUA EMAIL TAILIEUCHUANTHAMKHAO@GMAIL.COM

Tài liệu chuẩn tham khảo Phát triển kênh bởi Ths Nguyễn Thanh Tú Đơn vị tài trợ / phát hành / chia sẻ học thuật : Nguyen Thanh Tu Group Hỗ trợ trực tuyến Fb www.facebook.com/DayKemQuyNhon Mobi/Zalo 0905779594


CHUYÊN ĐỀ:

FI CI A

L

SỐ NGUYÊN TỐ VÀ MỘT SỐ BÀI TOÁN LIÊN QUAN CÁC KÍ HIỆU VÀ CỤM CHỮ CÁI VIẾT TẮT Kí hiệu

Tên tiếng Anh

Tên tiếng Việt

International Mathematical Olypiad

Kì thi Olimpic Toán quốc tế

VMO

Viet Nam Mathematical Olympiad

Kì thi học sinh giỏi quốc gia môn toán Việt Nam

IMO SL

International Mathematical Olypiad Shortlist

X TST

X Team Selection Tests

NMO

National Mathematical Olympiad

HSG

ƠN

Tuyển tập các bài toán trong danh sách rút gọn đề nghị cho kì thi Olimpic Toán quốc tế Kì thi chọn đội tuyển X

NH

TT & TT

OF

IMO

Tạp chí Toán học và tuổi trẻ Học sinh giỏi Học sinh giỏi quốc gia

QU Y

HSG QG

Kì thi học sinh giỏi toán quốc gia

Phần 1. ĐẶT VẤN ĐỀ

1. Lý do chọn đề tài

M

Số học là một phân nhánh toán học lâu đời nhất. Nó được hầu hết mọi người thường xuyên sử dụng trong những công việc thường nhật cho đến các tính toán khoa học.

DẠ

Y

Trong chương trình toán phổ thông, số học vốn là một phân môn khó. Các bài toán số học thường xuyên xuất hiện trong các kì thi học sinh giỏi các cấp và nó luôn là một thách thức đối với học sinh. Trong những bài toán số học ấy, chúng ta có thể nhận thấy rằng: các tính chất các ước nguyên tố thường xuyên được sử dụng một cách tinh tế và đẹp mắt. Tuy nhiên, hiện nay, các tài liệu viết riêng và chuyên sâu về nó không nhiều. Điều đó gây không ít khó khăn trong việc tiếp cận hướng giải quyết các bài toán số học đó.

1


FI CI A

L

Chính vì vậy, để giải quyết phần nào các khó khăn đó, chúng tôi chọn đề tài “Số nguyên tố và các bài toán liên quan” để trao đổi cùng các thầy cô và các em học sinh chuyên toán, góp phần nâng cao chất lượng giảng dạy môn toán trong trường THPT chuyên. Phần nội dung chuyên đề bao gồm ba chương:

- Chương 1. Các bài toán về ước nguyên tố. Nội dung của chương này xoay quanh các vấn đề về phân tích chính tắc và các định lý cổ điển liên quan đến ước nguyên tố của một số nguyên. - Chương 2. Thặng dư bậc hai.

OF

- Chương 3. Các bài toán về hệ số nhị thức và một số định lý khác. Nội dung này xoay quanh các tính chất ước nguyên tố trong hệ số nhị thức trong khai triển Newton và định lý Polignac.

ƠN

Trong các chương, hệ số ví dụ minh họa được đưa ra từ dễ đến khó theo quan điểm của chúng tôi. Trong mỗi ví dụ thường có định hướng, dẫn dắt tại sao có lời giải như vậy. Điều đó giúp cho việc học trở nên thuận lợi hơn. 2. Mục đích nghiên cứu

QU Y

NH

Đề tài “Số nguyên tố và các bài toán liên quan” được chúng tôi lựa chọn trao đổi cùng các đồng nghiệp về các tính chất, định lý về các số nguyên tố, ước nguyên tố của một số tự nhiên, sự phân tích chính tắc,... Vận dụng các định lý, tính chất đó giải quyết các bài toán số học trong các kì thi học sinh giỏi. Thông qua đề tài này, chúng tôi muốn nhấn mạnh, làm rõ tầm quan trọng của số nguyên tố trong các bài toán số học trong các kì thi học sinh giỏi Quốc gia và Quốc tế. 3. Nhiệm vụ nghiên cứu

M

Nghiên cứu các phương pháp giải bài toán số học trong các kì thi học sinh giỏi toán quốc tế, quốc gia Việt Nam và các nước trên thế giới. Cung cấp một tài liệu nhỏ về phương pháp dạy học phân môn số học cho các đồng nghiệp dạy chuyên toán, và phương pháp tự nghiên cứu cho học sinh chuyên. 4. Đối tượng và khách thể nghiên cứu

Đối tượng nghiên cứu của đề tài là học sinh chuyên Toán, đội tuyển học sinh giỏi quốc gia môn toán. 5. Phạm vi nghiên cứu

DẠ

Y

- Kiến thức trong phạm vi chương trình thi học sinh giỏi quốc gia của Bộ Giáo dục và Đào tạo.

- Nghiên cứu các đề thi học sinh giỏi toán quốc tế và học sinh giỏi toán quốc gia các nước. - Nghiên cứu các tài liệu số học có liên quan. 2


6. Phương pháp nghiên cứu

FI CI A

L

- Nghiên cứu lý luận: nghiên cứu các tài liệu, chương trình của Bộ Giáo dục và Đào tạo, các tài liệu số học hiện hành, các tạp chí toán học trong và ngoài nước, các tài liệu từ internet,… - Trao đổi, tọa đàm với các giáo viên chuyên và học sinh chuyên trong nước. - Tổng hợp, tổng kết kinh nghiệm.

Chương 1. Các bài toán về ước số nguyên tố

ƠN

1.1 Định nghĩa

OF

Phần 2. PHẦN NỘI DUNG

Một số nguyên dương p được gọi là số nguyên tố, nếu nó chỉ có hai ước số dương là 1 và chính nó.

NH

Nếu p không phải số nguyên tố thì p được gọi là hợp số. Nhận xét: 2 là số nguyên tố chẵn duy nhất. 1.2 Một số định lý, bổ đề cần dùng

1.2.1 Định lý 1 ( Định lý cơ bản của số học)

QU Y

Mọi số tự nhiên lớn hơn 1 đều có thể phân tích một cách duy nhất thành tích các thừa số nguyên tố. 1.2.2 Định lý 2 (Tính vô hạn của tập số nguyên tố) Tập hợp các số nguyên tố là vô hạn. 1.2.3 Định lý 3

M

Cho p là một số nguyên tố. Nếu p | ab thì p | a hoặc p | b.

1.2.4 Định lý 4 (Định lý Fermat nhỏ) Cho p là một số nguyên tố và a là một số nguyên sao cho (a, p) = 1. Khi đó

a p −1 ≡ 1(mod p) .

Y

Chứng minh

DẠ

Ta xét hệ thặng dư thu gọn mod p là A = {1, 2, …, p - 1}

Do (a, p) = 1 nên B = {a, 2a, …, (p - 1)a} cũng là hệ thặng dư thu gọn mod p.

Do đó ta có 1.2...( p − 1) ≡ a.2a...( p − 1)a (mod p ). 3


Mà p nguyên tố nên (1.2…(p - 1), p) = 1.

L

Từ đó ta có điều phải chứng minh.

FI CI A

1.2.5 Định lý 5 (Định lý Euler)

Cho a là một số nguyên, n là số nguyên dương, (a, n) = 1. Khi đó aϕ ( a ) ≡ 1(mod n) .

Chứng minh Ta xét hệ thặng dư thu gọn mod n là A = { a1 , a2 ,..., aϕ ( n ) }.

OF

Do (a, n) = 1 nên B = { a.a1 , a.a2 ,..., a.aϕ ( n ) } cũng là hệ thặng dư thu gọn mod n. Do đó ta có

a1.a2 ...aϕ ( n ) ≡ (a.a1 ).(a.a2 )...(aaϕ ( n ) ) ≡ aϕ ( a ) a1.a2 ...aϕ ( n ) (mod n)

1.2.6 Định lý 6 (Định lý Wilson) Cho p là một số nguyên tố lẻ. Khi đó

ƠN

Kết hợp với ( a1.a2 ...aϕ ( n ) ,n) = 1 ta dễ dàng suy ra đpcm.

Chứng minh (Xin nhường cho bạn đọc).

NH

( p − 1)! ≡ −1(mod p) .

QU Y

1.2.7 Bổ đề LTE

a) Cho p là số nguyên tố lẻ, x và y không chia hết cho p, x – y ⋮ p, n là một số nguyên dương. Khi đó

v p ( x n − y n ) = v p ( x − y ) + v p ( n).

M

b) Cho p là số nguyên tố lẻ, x và y không chia hết cho p, x + y ⋮ p, n là số nguyên dương lẻ. Khi đó v p ( x n + y n ) = v p ( x + y ) + v p (n).

c) Cho x, y lẻ, n là số nguyên dương chẵn. Khi đó

v2 ( x n − y n ) = v2 ( x 2 − y 2 ) + v2 ( n) − 1 .

Y

Chứng minh

DẠ

(Tham khảo “Phạm Văn Quốc – bổ đề LTE”) 1.2.8 Cấp phần tử 1.2.8.1 Các định nghĩa

Định nghĩa 1 4


L

Cho n > 1 và a là một số nguyên dương, (a, n) = 1. Số nguyên dương k nhỏ nhất thỏa mãn ak ≡ 1 (mod n) được gọi là cấp của a modulo n. Kí hiệu k = ordn(a).

FI CI A

Định nghĩa 2

Cho n > 1 và a là một số nguyên dương, (a, n) = 1. Nếu ϕ(n) = ordn(a) thì a được gọi là một căn nguyên thủy modulo n. Nhận xét: Từ định nghĩa trên ta dễ dàng suy ra

1.2.8.2 Các định lý Định lý 1

ƠN

Cho a, n thỏa mãn n > 1, (a, n) = 1. Khi đó

OF

+) Nếu a là căn nguyên thủy ( mod n) thì mọi số cùng lớp với a theo (mod n) đều là căn nguyên thủy (modn).

ax ≡ 1 (mod n) ⇔ x ⋮ ordn (a). Chứng minh Đặt k = ordn (a).

Theo thuật toán Euclid ta có

NH

Giả sử ax ≡ 1 (mod n).

x = kq + r, 0 ≤ r < k.

QU Y

Khi đó 1 ≡ ax ≡ (ak)qar ≡ ar (mod n).

Suy ra ar ≡ 1 (mod n)  r = 0 (theo định nghĩa). Vậy x ⋮ k.

Chiều ngược lại hiển nhiên.

M

Hệ quả

Cho a, n thỏa mãn n > 1, (a, n) = 1. Khi đó ϕ(n) ⋮ ordn (a).

Định lí 2

Y

Nếu a là căn nguyên thủy (mod n) thì tập A = {1, a, a2,…,ah-1} là hệ thặng dư thu gọn (mod n) (lúc này h = φ (n))

DẠ

Định lí 3

Nếu p là một số nguyên tố thì có đúng φ (p - 1) căn nguyên thủy (mod p)

Định lít 4 5


L

Nếu p là một số nguyên tố lẻ và a là một căn nguyên thủy (mod p2) thì a cũng là căn nguyên thủy (mod pn) với n ≥ 3.

FI CI A

Định lý 5 (Định lý về sự tồn tại căn nguyên thủy)

Cho m là một số nguyên, m > 1 khi đó m có căn nguyên thủy khi và chỉ khi m có một trong 4 dạng sau: 2, 4, pα, 2p α (trong đó p là 1 số nguyên tố lẻ) (Phần chứng minh các định lí trên, xin nhường cho bạn đọc). 1.3 Các ví dụ điển hình

OF

Trong mục này, để thuận lợi cho công tác giảng dạy, dựa vào độ phức tạp các bài toán, tôi sẽ chia thành hai phần. Phần một là các ví dụ cơ bản, giúp học sinh làm quen dần với các dạng toán. Phần hai là các bài toán nâng cao, đòi hỏi kĩ năng xử lí tinh tế và được cập nhật trong các kì thi học sinh giỏi gần nhất.

ƠN

1.3.1 Các ví dụ cơ bản

NH

Trong phần này, tôi sẽ đưa ra các ví dụ điển hình về tính chất ước nguyên tố. Vận dụng các tính chất đó vào giải quyết các bài toán liên quan. Tuy nhiên, các ví dụ được chọn không quá khó, nhưng thường xuyên xuất hiện. Cố gắng giúp học sinh hình thành, phát triển tư duy trong phân môn số học. Ví dụ 1 (Đề tham khảo 30/4/2021) Giả sử a, b, c là các số nguyên dương thỏa mãn hệ phương trình

QU Y

 21a = 43ϕ (b)   21b = 43ϕ (c)  21c = 43ϕ (a ) 

Chứng minh rằng a = b = c.

M

Nhận xét: đây là một dạng của hệ hoán vị vòng quanh nên việc giải quyết hệ này thông thường ta xét a ≤ b ≤ c sau đó chứng minh a = b = c.

Mặt khác, bài toán liên quan đến phi hàm Euler nên thông thường ta sử dụng phân tích chính tắc để đánh giá là một hướng đi chung. Mời các bạn theo dõi lời giải sau đây. Lời giải

Ta có (21, 43) = 1  a, b, c cùng chia hết cho 43. Do đó, a, b, c

DẠ

Y

Xét phân tích chính tắc của a, b, c như sau n

α

m

α

k

α

a = ∏ pai ai , b = ∏ pbi bi , c = ∏ pci ci i =1

i =1

Khi đó 6

i =1

≥ 43.


α ai −1 i

i =1

( pai − 1)⋮ (43 − 1)⋮ 2

L

n

ϕ ( a ) = ∏ pa

FI CI A

Chứng minh tương tự ta được ϕ (a), ϕ (b), ϕ (c) chẵn. Do (21, 2) = 1  a ⋮ 2, b ⋮ 2. c ⋮ 2. Từ đó ta có 21a = 43ϕ (b) = 43.b∏ (1 − i =1

1 1 1 ) ≤ 43.b(1 − )(1 − ) = 21b pbi 2 43

≤ b.

a

Chứng minh tương tự ta được b

OF

m

≤ c; c ≤ a.

Từ đó suy ra a = b = c.

Ví dụ 2 (Chọn ĐT 10 KHTN 2020) Tìm tất cả các số nguyên tố p sao cho

ƠN

3p + 5p – 1 là số nguyên tố.

Lời giải

TH1: p = 3k + 1, (k chẵn) ta có

NH

Nhận xét: Bài toán này khá đơn giản. Trước tiên ta thử nghiệm, dự đoán p = 3, sau đó xét p khác 3 và chứng minh 3p + 5p – 1 không là số nguyên tố. Lời giải cụ thể như sau:

QU Y

3p + 5p – 1 = 3.27k + 5.125k – 1

3(-1)k + 5.(-1)k – 1

0 (mod 7).

2(-1)k + 4(-1)k – 1

0 (mod 7).

Nên 3p + 5p – 1 không là số nguyên tố. TH2: p = 3k + 2, (k lẻ) ta có

3p + 5p – 1 = 9.27k + 25.125k – 1 Vậy p = 3k

 p = 3.

M

Thử lại thấy p = 3 thỏa mãn.

Ví dụ 3 (Chọn ĐT 10 KHTN 2020) Tìm x, y nguyên dương sao cho 2x + 5y + 2 là số chính phương. Nhận xét: Bài này cũng giống như bài trên, Tính chất chủ đạo sử dụng ở đây là mọi ước của pn đều có dạng pk, 0 ≤ k ≤ n. Cụ thể ta có lời giải sau:

Y

Lời giải

DẠ

Xét x

≥ 2 ta có 2x + 5y + 2 ≡ 3 (mod 4)  2x + 5y + 2 không là số chính phương.

Do đó x = 1.

Khi đó ta đặt 5y + 4 = n2 ⇔ (n -2)(n + 2) = 5y. 7


FI CI A

Từ đó tìm được (x, y) = (1, 1).

L

n − 2 = 5a  5b − 5a = 4  a = 0, b = 1 . Đến đây ta suy ra  b n + 2 = 5

Ví dụ 4 (Olimpic KHTN 2014) Tìm tất cả bộ ba số (x, n, p) với x, n nguyên dương, p nguyên tố thỏa mãn

x3 + 2 x = 2( p n − 1) . Lời giải

OF

Ta có

x3 + 2 x = 3( p n − 1) ⇔ x3 + 2 x + 3 = 3 p n ⇔ ( x + 1)( x 2 − x + 3) = 3 p n .

ƠN

Do d = (x + 1, x2 – x + 3) = (x + 1, (x+1)(x-2) + 5) = (x +1, 5).

NH

 x + 1 = 3  2 n  x − x + 3 = p 2 Nếu d = 1, do x + 1 > 1 và x –x + 3 > 1 nên  n   x + 1 = p   x 2 − x + 3 = 3. 

Từ đó tìm được (x, n, p) =(2, 2, 2) và (x, n, p) = (1, 1, 2). Nếu d = 5 thì x + 1 = 5m.

QU Y

Thay vào phương trình ban đầu ta được 5m(25m 2 − 15m + 5) = 3 p n  p = 5  m(5m 2 − 3m + 1) = 3.5n −2.

Nếu m = 1 thì tính được n = 2, x = 4  (x, n, p) = (4, 2, 5).

M

Nếu m ≥ 2 thì 5m2 – 3m + 1 > 3.

Do d = (m,5m2 – 3m + 1) = 1  m = 3 và 5m2 – 3m + 1 = 5n – 2 vô nghiệm.

Vậy ta tìm được ba bộ (x, n, p) như trên. Ví dụ 5 (Olimpic KHTN 2015) Tìm tất cả các số nguyên tố p sao cho 3p + 4p là số chính phương.

Y

Lời giải

DẠ

Ta có 3p + 4p = x2  (x – 2p) (x + 2p) = 3p

8


FI CI A

Do đó 2p + 1 = 3p – 1  p = 2 (thỏa mãn)

L

 x + 2 p = 3 p −n  2 p +1 = 3n (3 p −2 n − 1)  n = 0 .   p n  x − 2 = 3

Ví dụ 6 (TH & TT_ T10/507) Tìm số nguyên tố p và hai số nguyên dương a, b sao cho pa + pb là số chính phương. Lời giải

TH1: a = b suy ra 2.pa = x2  p = 2, a lẻ  (p, a, b) = (2, 2n + 1, 2n + 1), n ∈ ℕ .

OF

TH2: a ≠ b. Không mất tính tổng quát ta giả sử a > b  pb(pa – b + 1) = x2. Mà (p , p

a–b

b 2  p = u + 1) = 1 nên  a −b 2  p + 1 = v .

ƠN

b

Do đó b chẵn. Ta có

NH

pa – b = (v – 1)(v + 1).

Nếu v = 2 thì pa – b = 3  a – b = 1 và p = 3  (p, a, b) = (3, 2k + 1, 2k). Nếu v > 2 thì p | v – 1 và p | p + 1  p | 2  p = 2  2a – b = (v - 1)(v + 1).

Suy ra a – b = 3.

QU Y

v − 1 = 2k  2l − 2k = 2  k = 1, l = 2  v = 3 .   l v + 1 = 2

Từ đó suy ra (p, a, b) = (2, 2k + 3, 2k). Kết luận… n

M

Ví dụ 7 (TH&TT_ T10/512) Chứng minh rằng với mọi n nguyên dương thì ta luôn có

n và 2 2 + 1 là hai số nguyên số nguyên tố cùng nhau.

Lời giải

n

Y

Gọi p là số nguyên tố và p | 2 2 + 1 thì theo một tính chất quen thuộc về bậc của một số nguyên ta có p ≡ 1 (mod 2n + 1).  p = 2n + 1k + 1, k ∈ ℕ *  p > 2n + 1.

DẠ

Mặt khác, ta dễ chứng minh bằng quy nạp được 2n + 1 > n, với mọi n ∈ ℕ *. Từ đó ta có đpcm.

Ví dụ 8 (TH&TT_T11/513) Cho h là số nguyên dương sao cho p = 2h + 1 là một số nguyên tố. Tìm bậc của 2 modulo p. 9


L

Nhận xét: đây là bài toán cơ bản nhất mà sau khi dạy lý thuyết về cấp phần tử và có thể lấy làm ví dụ minh họa mở đầu.

FI CI A

Lời giải

Ta có 2 h ≡ −1(mod p )  2 2 h ≡ 1(mod p ). Đặt k = ord p (2)  k | 2h nhưng k không là ước của h.

N ếu k

≤ h thì 2k – 1 < 2h + 1 = p (loại)

 k > h.

Nếu k lẻ mà k | 2h  k | h (vô lý).

≤ h.

(1)

OF

Vậy k chẵn  k = 2t | 2h  t | h  t Ta có

2k – 1 = 22t – 1 = (2t - 1)(2t + 1).

≤ h  2t – 1 < 2h + 1 = p  (p, 2t - 1) = 1.  2t + 1 ⋮ p  2t + 1 ≥ p = 2h + 1  t ≥ h. (2)

ƠN

Do t

Từ (1) và (2) suy ra t = h  k = 2h.

NH

Ví dụ 9 (HSG Tỉnh THPT Phú Yên 2021) Cho p là số nguyên tố lẻ. a và b là hai số nguyên dương lẻ sao cho a + b chia hết cho p và a – b chia hết cho p – 1. Chứng minh rằng

ab + ba ⋮ 2 p

QU Y

Nhận xét: Bài này có hình thức khá đẹp mắt mà tôi rất thích. Nó thực chất đã xuất hiện khá lâu, trên TH&TT và chuyên đề cấp phần tử (2014) của thầy Nguyễn Duy Liên. Lời giải

Ta có a, b lẻ nên ab + ba chẵn nên ab + ba ⋮ 2 Không mất tính tổng quát ta giả sử a

≥ b

M

Ta có a a − a b = a b (a a −b − 1) = a b ( a k ( p −1) − 1) ≡ 0(mod p )  a a ≡ a b (mod p )

Do b lẻ nên a a + b a ⋮ a + b⋮ p  a a + b a ≡ 0(mod p ) Từ trên ta có a b + b a ≡ a a + b a ≡ 0(mod p ) Từ đó ta có đpcm.

DẠ

Y

Ví dụ 10 (TH&TT_ T11/472) Cho n, k là các số nguyên dương và p là số nguyên tố lẻ. Chứng minh rằng tồn tại số nguyên dương a sao cho pk là ước của

ϕ (a), ϕ (a + 1), …, ϕ (a + n).

10


L

Nhận xét: Những bài chứng minh sự tồn tại hoặc tồn tại vô hạn các số nguyên dương thỏa mãn điều kiện nào đó, ta thường nghĩ đến định lý Dirichle hoặc xây dựng hệ phương trình đồng dư sau đó áp dụng định lý thặng dư Trung Hoa.

FI CI A

Lời giải

Gọi q0, q1, …, qn là các số nguyên tố sao cho qi ≡ 1 (mod pk), điều này tồn tại theo bài toán trên.

 a ≡ 0(mod q0 )   a ≡ −1(mod q1 )  ...  a ≡ − n(mod qn ) Hay a + i ≡ 0 (mod qi).

ƠN

m

OF

Theo định lí Thặng dư Tru ng Hoa tồn tạo số nguyên dương a sao cho

Mặt khác ta lại có ϕ ( p1 ... pm ) = ∏ piαi −1 ( pi − 1). α1

αm

i =1

NH

 Nếu p | n thì p – 1 | ϕ (n)

Từ đó suy ra pk | qi – 1 | ϕ (a + i), i = 0, 1, 2, …, n.

Đó chính là đpcm.

Lời giải

QU Y

Ví dụ 11 (Pi 233) Cặp số nguyên dương a, b được gọi là cân bằng nếu hai số nguyên dương a, b có cùng tập ước nguyên tố. Tìm tất cả các số nguyên dương n sao cho n và n + 6 là cặp cân bằng. Giả sử n là số nguyên dương sao cho (n, n + 6) là cặp cân bằng. Dễ thấy n > 1.

Do đó

M

Gọi p là ước nguyên tố bất kì của n

 p | (n, n + 6)  p ∈ {2, 3}.

 n = 2a 3b  2u 3v − 2a 3b = 6. (1)  u v  n + 6 = 2 3

TH1: a = 0 thì u = 0  3v – 3b = 6 ⇔ 3b(3v – b - 1) = 6.

Y

 n = 3 (thỏa mãn). TH2: Nếu b = 0  v = 0  2u – 2a = 6  a = 1, u = 3  n = 2 (thỏa mãn).

DẠ

Từ đó dễ dàng tìm được b = 1, v = 2

TH3: a,b, u, v > 0 thì 2u – 1.3v – 1 – 2a – 1.3b – 1 = 1.

11


Lại có u và v không đồng thời bằng 1 nên chỉ có thể xảy ra các khả năng sau:

Phương trình này quen thuộc và dễ dàng giải được a ∈ {2, 4} +) a = v = 1  2u – 1 – 3b – 1 = 1  b ∈ {1, 2}  n ∈ {6, 18}. Từ đó ta tìm được n ∈ {2, 3, 6, 12, 18, 48 }.

FI CI A

 2u – 1.3v – 1 = 2  v = 1, u = 2  n = 6 (thỏa mãn). +) b = u = 1  3v – 1 – 2a – 1 = 1. +) a = b = 1

L

Do đó trong hai số u và a phải có một số bằng 1, trong hai số v và b phải có một số bằng 1.

 n ∈ {12, 48}.

ƠN

1 1 1 + + ... + =1 a1 a2 an

OF

Ví dụ 12 (TH&TT - 249) Giả sử a1, a2, …, an là các số nguyên dương phân biệt thỏa mãn

và số lớn nhất trong các số ai có dạng 2p với p là một số nguyên tố. Hãy xác định các số a1, a2, …, an.

NH

Lời giải

Không mất tính tổng quát ta giả sử a1 = max {a1, a2, …, an} = 2p với p nguyên tố. Ta có

QU Y

1 1 1 1 1 2 p −1 + + ... + = 1 − = 1 − = a2 a3 an a1 2p 2p

m 2 p −1 = ⇔ 2 pm = (2 p − 1).a2 a3 ...an a2 a3 ...an 2p

Do đó p| a2 a3 ...an (vì (p. 2p - 1) = 1).

Suy ra p là ước của một trong các số ai, i = 2, 3, …, n.

M

Không mất tính tổng quát, giả sử p | a2.

Mà a1 = 2p > a2  a2 = p.

DẠ

Y

Từ đó suy ra

1 1 1 1 1 1 1 2p −3 + + ... + = 1 − − = 1 − − = a3 a4 an a1 a2 2p p 2p ⇔

M 2p −3 = ⇔ 2 pM = (2 p − 3).a3a4 ...an . a3a4 ...an 2p

Suy ra p | (2 p − 3).a3 a4 ...an .

Nếu có số ai chia hết cho p (với i = 3, 4, …, n) thì ai = p = a2 (vô lý) nên p | 2p – 3 12


 p = 3.

L

Vậy số lớn nhất a1 = 6, a2 = 3.

FI CI A

Các số còn lại nhỏ hơn 6 nên thuộc tập {1, 2, 4, 5}. Từ đó dễ dàng chứng minh được tồn tại duy nhất bộ {2, 3, 6} thỏa mãn.

1.3.1 Các bài toán nâng cao

OF

Trong phần này, chúng tôi cố gắng đưa ra các ví dụ minh họa mang tính thời sự và tinh tế hơn. Các hướng giải quyết phức tạp hơn (theo quan điểm cá nhân)

Ví dụ 13 (IMO SL 2018) Tìm tất cả các cặp số nguyên dương (n, k) phân biệt sao cho tồn tại số nguyên dương s sao cho số ước nguyên dương của sn và sk là bằng nhau.

ƠN

Lời giải (Dựa theo lời giải của v_Enhance trên Aops) TH1: Nếu n | k hoặc k | n thì hiển nhiên không tồn tại s. TH2: n và k không có số nào là bội của số kia.

Nhận xét rằng: với a > b không âm c sao cho

NH

Ta sẽ chứng minh luôn tồn tại số nguyên dương s sao cho số ước của sn và sk là bằng nhau.

≥ 0 và vọi M nguyên, M ≥ b + 1 luôn tồn tại số nguyên

QU Y

a + c +1 M +1 = . b + c +1 M

Thật vậy, ta chỉ việc chọn c = M (a − b) − b + 1 ≥ (b + 1)(b + 1 − b) − b + 1 > 0 , luôn thỏa mãn đẳng thức trên. Bây giờ ta xét

β

M

n = p1α1 ... piαi .q1β1 ...q j j .r1γ 1 ...rmγ m '

'

β

'

'

k = p1α1 ... piα i .q1β1 ...q j j .r1γ 1 ...rmγ m .

Với α l > α l' ,(l = 1, 2,..., i ), γ s < γ s' ,( s = 1, 2,..., m), i, m ≥ 1, j ≥ 0. Ta chọn M > max{α1 ,...,α i , γ 1' ,..., γ m' } . Khi đó theo nhận xét trên ta luôn tìm được các

DẠ

Y

số λ1 ,..., λi sao cho i

λs + α s + 1 iM + 1 iM + 2 iM + i M +1 = . ... = . ' iM iM + 1 iM + i − 1 M s + αs +1

∏λ s =1

Và các số λ1' ,..., λm' sao cho 13


λs' + γ s + 1 mM + 1 mM + 2 mM + m M +1 = . ... = ∏ ' ' mM mM + 1 mM + m − 1 M s =1 λs + γ s + 1 '

L

m

'

FI CI A

Chọn s = p1λ1 ... piλi .r1λ1 ...rmλm thì ta sẽ có i

λs + α s + 1 d ( sn) s =1 s + α s' + 1 M + 1 M = m ' = . = 1. λs + γ s + 1 d ( sk ) M M +1 ∏ ' ' s =1 λs + γ s + 1

∏λ

Vậy các cặp (s,k) mà không có số nào là bội của số còn lại luôn thỏa mãn bài toán.

OF

Ví dụ 14 (Turkey EGMO TST 2020) Kí hiệu p ( m ) là số các ước nguyên tố phân

 p ( m) + 1   2 

biệt của số nguyên dương m > 1 . Đặt f ( m ) là ước nguyên tố nhỏ thứ 

ƠN

của m . Tìm tất cả các số nguyên dương n thỏa mãn đẳng thức sau:

f ( n 2 + 2 ) + f (n 2 + 5) = 2n − 4 . Lời giải

NH

Vì mọi ước nguyên tố đều lớn hơn hoặc bằng 2 nên:

2n − 4 = f ( n 2 + 2 ) + f ( n 2 + 5) ≥ 2 + 2  n ≥ 4 .

QU Y

Với n = 4 : f (18) + f (21) > 4 (loại).

Với n = 5 : f (27) + f (30) = 3 + 3 = 6 = 2.5 − 4 (thỏa mãn). Với n = 6 : f (38) + f (41) > 8 (loại).

Với n = 7 : f (51) + f (54) = 3 + 2 < 10 (loại).

M

Ta chứng minh bổ đề: nếu n là một hợp số thì f ( n) ≤ n .

Xét hai trường hợp:

pt = n .

Nếu n = p t với t ∈ ℕ, t ≥ 2 . Khi đó p ( n ) = 1 và f (n) = p <

 p (n) + 1  < p ( n) nên ước nguyên tố thứ p ( n ) của  2 

n cũng lớn hơn

n , vô lý vì n không thể có hai ước nguyên tố phân biệt cùng lớn hơn

DẠ

Y

Nếu p ( n ) ≥ 2 mà f ( n) > n , vì 

n.

Bổ đề trên được chứng minh. Quay lại bài toán. 14


chỉ xét n ≥ 8 và sử dụng bổ đề ta có

f ( n 2 + 5) ≤ n 2 + 5  f ( n 2 + 5 ) ≤ n . Nếu f ( n 2 + 2) = n thì n | n 2 + 2  n | 2  n = 2 (vô lý).

FI CI A

L

f ( n2 + 2) ≤ n2 + 2  f ( n2 + 2 ) ≤ n

Tương tự nếu f (n 2 + 5) = n thì n | n 2 + 5  n | 5  n = 5 (vô lý).

OF

Nếu f (n 2 + 2) = n − 1 thì n − 1| n 2 + 2  n − 1| 3  n ≤ 4 (vô lý).

Tương tự nếu f (n 2 + 5) = n − 1 thì n − 1| n 2 + 5  n − 1| 6  n ≤ 7 (vô lý).

(

)

ƠN

Suy ra f n 2 + 5 ≤ n − 2 (1).

Nếu f ( n 2 + 2) = n − 2 thì n − 2 | n 2 + 2  n − 2 | 4  n ≤ 6 (vô lý).

(

)

(

)

NH

Do đó f n 2 + 2 ≤ n − 3 (2).

Từ (1) và (2) suy ra f n 2 + 2 + f (n 2 + 5) ≤ 2n − 5 < 2n − 4 (loại). Vậy n = 5 là giá trị duy nhất thỏa mãn đề bài.

QU Y

Ví dụ 15 (IMO Shortlist 2020) Cho trước số nguyên dương k . Chứng minh rằng tồn tại số nguyên tố p sao cho có thể chọn các số nguyên dương phân biệt

a1 , a2 ,..., ak +3 ∈ {1;2;...; p − 1} thỏa mãn ai ai +1ai + 2 ai +3 − i chia hết cho p với mọi

i = 1,2,..., k . Lời giải

k +3

M

Trước tiên ta xây dựng dãy số hữu tỉ {qn }n =1 như sau:

1  2 q1 = 1; q2 = M ; q3 = M ; q4 = M 3 .  n + 1 * q = qn∀n ∈ ℕ  n+ 4 n

DẠ

Y

Trong đó M là một số hữu tỉ tùy ý thỏa mãn điều kiện sau:

1   1  1  1   M >  1 +  1 +  1 +  ... 1 + .  1  2  3   k + 3 

Sử dụng quy nạp ta dễ dàng chứng minh được qn qn +1qn + 2 qn +3 = n ∀n = 1;2;...; k . 15


k +3

Bây giờ ta chứng minh các số hạng của dãy {qn }n =1 đôi một phân biệt. Từ cách xây k +3

L

dựng dãy số {qn }n =1 ta thấy ngay:

FI CI A

q1 < q5 < q9 < ... q < q < q < ...  2 6 10  q3 < q7 < q11 < ... q4 < q8 < q12 < ... Nhận thấy:

k +3

ƠN

OF

1  1  1  Minq4 n+1 = q1 = 1 = q4 .M 3 > q4 .M > q4 1 + 1 +  1 +  ... > Max{q4 n }  4  8  12   1  1  1  Minq4 n+2 = q2 = M > q1 1 + 1 + 1 +  ... > Max{q4 n+1} .  1  5  9  1  1  1  Minq4 n+3 = q3 = M 2 = q2 .M > q2 1 + 1 + 1 +  ... > Max{q4 n+2 }  2  6  10 

k +3

NH

Do đó các số hữu tỉ trong dãy {qn }n =1 là đôi một khác nhau. Tiếp theo ta chọn số nguyên tố p và xây dựng {an }n =1 như sau:

rn rn , sn ∈ ℕ* ;(rn ; sn ) = 1) ∀n = 1;2;...; k + 3 . ( sn

QU Y

Đặt qn =

Khi đó tồn tại số nguyên tố p đủ lớn thỏa mãn:

 p > Max rn , p > Max sn .  p > Max r s − r s i j j i  1≤i ≠ j ≤ k +3

M

Ta có ngay: rn ⋮ p, s n ⋮ p ∀n; ri s j − rj si ⋮ p ∀i ≠ j .

Tiếp theo, với mỗi n = 1;2;...; k + 3 ta chọn an ∈ {1;2;...; p − 1} sao cho an ≡ rn sn−1 (mod p ), trong đó sn−1 là phần tử nghịch đảo của sn trong {1;2;...; p − 1} . Điều này là khả thi do rn ⋮ p, s n ⋮ p . Ta đi chứng minh

Y

{an } thỏa mãn bài toán.

DẠ

+ ∀i ≠ j thì ai ≠ a j . −1 −1 Thật vậy giả sử ai = a j  ri si ≡ rj s j (mod p)  ri s j ≡ rj si (mod p ) (vô lý).

Do đó ta có 16


ai ai +1ai + 2 ai +3 ≡ ri ri +1ri + 2 ri +3 si−1si−+11si−+12 si−+13 ≡ i (mod p ) Đó chính là điều phải chứng minh.

FI CI A

L

 ai ai +1ai + 2 ai +3 − i⋮ p. Ví dụ 16 (Centroamarican 2020 – Pr 6) Ta gọi một số nguyên dương N là số Inteoceanic nếu nó có phân tích chính tắc

N = p1x1 . p2x2 ... pkxk thỏa mãn

Hãy tìm tất cả các số Inteoceanic nhỏ hơn 2000. Lời giải

Thật vậy, nếu k

≤ 3.

ƠN

Trước hết ta chứng minh k

OF

p1 + p2 + ... + pk = x1 + x2 + ... + xk .

≥ 4 thì

p1 + p2 + ... + pk ≥ 2 + 3 + 5 + 7 = 17

NH

 N > 2 x1 2 x2 ...2 xk ≥ 2 x1 + x2 +...+ xk ≥ 217 > 2000 không thỏa mãn. V ậy k

≤ 3.

Với k = 3 ta có

QU Y

N = p1x1 p2x2 . p3x3 > 2 x1 2 x2 .4 x3 = 2 x1 + x2 + 2 x3 ≥ 22+3+5+ x3 ≥ 211 > 2000 (loại). Với k = 2 ta có p1 + p2

≤ 10 nên chỉ có các cặp (p1, p2) sau (2, 3), (2, 5), (2, 7), (3, 5), (3, 7).

TH1: x1 + x2 = 2 + 3 = 5 ta có các nghiệm sau

M

N ∈ {24.3, 23.32, 22.33, 21.34}.

TH2: x1 + x2 = 2 + 5 = 7, ta có 54.23 > 2020 nên ta tìm được N ∈ {53.24, 52.25, 5.26}.

TH3: x1 + x2 = 2 + 7 ta dễ dàng tìm được nghiệm duy nhất N = 28.7 TH4: x1 + x2 = 3 + 5 = 8.

Y

Ta có 37.5 > 2000 nên dễ dàng chứng minh được không tồn tại N.

DẠ

TH5: x1 + x2 = 3 + 7 = 10 cũng dễ chứng minh được không tồn tại N.

Với k = 1 dễ dàng tìm được N ∈ {22, 33}.

Từ đó tìm được tất cả các số Inteoceanic thỏa mãn bài toán như trên. 17


Ví dụ 17 (BMO 2019) Kí hiệu P là tập hợp gồm tất cả các số nguyên tố.

v ớ i mọ i p , q ∈ P . Lời giải

Từ đẳng thức đã cho ta có

f ( p ) f ( q ) − f ( q) f ( p ) = p q − q p .

FI CI A

f ( p) f ( q ) + q p = f (q) f ( p ) + p q

L

Tìm tất cả các hàm f : P → P thỏa mãn đẳng thức sau

OF

Với p, q là các số nguyên tố lẻ bất kì thì p q − q p là số chẵn nên dễ thấy f ( p ), f ( q ) phải cùng tính chẵn lẻ.

Giả sử f ( p ), f ( q ) đều chẵn thì do f : P → P nên f ( p ) = f ( q ) = 2 .

ƠN

Thay vào ta được q p = p q , vô lý.

Do đó với mọi số nguyên tố lẻ p thì f ( p ) lẻ.

NH

Thế q = 2 và p là một số nguyên tố lẻ bất kì vào đẳng thức đã cho ta được

f ( p ) f (2) − f (2) f ( p ) = p 2 − 2 p . Vì vế phải là một số lẻ nên f ( p ) và f (2) khác tính chẵn lẻ. Lại có f ( p ) lẻ nên

QU Y

f (2) = 2 .

Bây giờ xét p là một số nguyên tố lẻ bất kì: f ( p ) 2 − 2 f ( p ) = p 2 − 2 p . Xét hàm số g ( x) = x 2 − 2 x . Ta chứng minh g ( x + 1) < g ( x ) với mọi x ≥ 3. Thật vậy:

2

M

g ( x + 1) − g ( x) = ( x + 1) − 2x+1 − ( x2 − 2x ) = 2 x + 1 − 2x < 0 với mọi x ≥ 3.

Do đó từ đẳng thức

f ( p) 2 − 2 f ( p ) = p 2 − 2 p .

ta thu được f ( p ) = p với mọi số nguyên tố lẻ p .

Y

Kết luận: hàm số cần tìm là hàm f ( p ) = p (thử lại thấy thỏa mãn).

DẠ

Ví dụ 18 (Canada TST 2021) Cho n là một số nguyên dương. Gọi p1, p2, …, pn là các số nguyên tố phân biệt lớn hơn 3. Chứng minh rằng 2 p p ... p + 1 có ít nhất 4n ước số. 1 2

n

Lời giải Bổ đề 1. Cho a, b là hai số nguyên dương lẻ. Khi đó (2a + 1, 2b + 1) = 2(a, b) + 1. 18


Đây là bổ đề khá quen thuộc, xin không chứng minh lại ở đây.

 2p + 1 không chia hết chi 9.

FI CI A

Nếu (p, 3) = 1 thì theo bổ đề trên ta có (2p + 1, 23 + 1) = 2(p,3) + 1 = 3

L

Trở lại bài toán

Ta gọi một số nguyên dương là “tốt” nếu nó là số Squarefree lẻ, không chia hết cho 3 và lớn hơn hoặc bằng 5. Đặt T(n) là số ước nguyên tố phân biệt của n, d(n) là số ước của n.

d (2a + 1) ≥ 4T ( a ) (*) Quy nạp theo T(a).

≥ 5, p nguyên tố.

Khi đó 2a + 1 = (2 + 1)(2a−1 + ... + 1) .

ƠN

Nếu T(a) = 1 hay a = p

OF

Bài toán trở thành: Chứng minh rằng với mọi số tốt a ta có

Mà 2a−1 + ... + 1 > 3 , lẻ và không chia hết cho 3 nên nó có ít nhất một ước nguyên tố lớn hơn hoặc bằng 5

Do đó (*) đúng khi T(a) = 1.

NH

 d (2a + 1) ≥ 4 = 4T ( a ).

Giả sử (*) đúng khi T(a) = k, ta xét số nguyên tố ( p, a) = 1, p ≥ 5, p là số nguyên tố.

QU Y

Khi đó

pa p 2 + 1⋮ 2 + 1  2 pa + 1 = L[2a + 1, 2 p + 1] .  pa a 2 + 1⋮ 2 + 1

Do Nên

≥ 3.3 – 8 > 0

M

pa - 2p - 2 - 4 = (p - 2)(a - 2)-8

2 pa + 1 > 2 2 p + 2 a + 4 + 1 > (2a + 1) 2 (2 p + 1) 2 > [2a + 1, 2 p + 1]2

Suy ra L > [2 a + 1, 2 p + 1] =

(2 a + 1)(2 p + 1) 3

DẠ

Y

Suy ra d ( L[2a + 1, 2 p + 1]) ≤ 2d ([2a + 1, 2 p + 1]) = 2.

Từ đó (*) đúng với T(a) = k + 1. Vậy ta có đpcm. 19

d (2a + 1) d (2 p + 1) ≥ 4T ( a ) +1 2


L

Ví dụ 19 (Hướng tới kì thi VMO và Olimpic Toán) Cho p là số nguyên tố và a, b, c là 3 số nguyên bất kì. Chứng minh rằng tồn tại các số nguyên x, y, z không đồng thời chia hết cho p sao cho

FI CI A

ax 2 + by 2 + cz 2 chia hết cho p. Lời giải

(Tham khảo từ một bạn trên Group Hướng tới kì thi VMO và Olimpic Toán)

TH1: Nếu một trong ba số a, b, c chia hết cho p thì bài toán hiển nhiên đúng.

OF

Bởi vì, chẳng hạn c ⋮ p, ta chỉ việc chọn (x, y, z) = (p, p, 1) thỏa mãn.

TH2: Cả ba số a, b, c cùng không chia hết cho p, chọn z = 1. Bài toán trở thành ax 2 + by 2 ≡ −c(mod p )

ƠN

có nghiệm.

p +1 p +1 thặng dư bình phương (mod p) nên ax 2 , −c − by 2 đều có số dư khi 2 2 chia cho p khi x và y chạy khắp ℤ .

NH

Do có

Nếu các số dư của ax2 luôn khác số dư của –c – by2 khi chia cho p thì vô lý vì khi đó có p + 1 số dư phân biệt. Do đó phải tồn tại x, y ∈ Z sao cho ax 2 ≡ −c − by 2 (mod p ) .

QU Y

Đến đây ta có điều phải chứng minh.

Nhận xét: Thực chất, bài toán này chỉ là một hệ quả trực tiếp của định lý Cauchy – Davenport! Ví dụ 20

≥ 1 và p là số nguyên tố với x0 ≡/ 1(mod p ) . Đa thức

M

a) Cho x0, k ∈ ℤ , k

P ( x) = 1 + x p

k −1

+ x2 p

k −1

k −1

+ ... + x ( p −1). p .

Giả sử q là một ước nguyên tố của P(x0), chứng minh rằng q ≡ 1 (mod pk). b) Chứng minh rằng tồn tại vô số số nguyên tố dạng pkt + 1. Lời giải

k

Y

a) Ta có P ( x0 ) | x0p − 1.

DẠ

Gọi h = ord q ( x0 )  h | pk  h = pn với n

N ếu n

k −1

≤ k – 1 thì x0p ≡ 1(mod q)

≤ k.

 P(x0) ≡ p (mod q)

Mà P(x0) ≡ 0 (mod q)  p = q 20


Mà theo định lý Fermat nhỏ x0p ≡ x0 (mod p )  x0p

k −1

≡ x0 (mod p )

Do đó n = k tức là h = pk. Từ đó suy ra q – 1 ⋮ pk  q ≡ 1 (mod pk). b) Giả sử có hữu hạn số nguyên tố dạng pkt + 1 là q1, q2, …, qm. Xét x0 = q1.q2…qm + 1 Ta có x0 ≡ 1 + 1 ≡ 2 (mod p)  x0 ≡/ 1(mod p )

FI CI A

L

Do đó x0 ≡ 1 (mod p) trái với giả thiết

Do đó, tồn tại qi, i = 1, 2, …, m sao cho p = qi.

OF

Gọi q là ước nguyên tố của P(x0), q luôn tồn tại vì P(x0) > 1, theo phần a) thì ta có q ≡ 1(mod p k ) .

ƠN

Suy ra x0 ≡ 1 (mod q)  P(x0) ≡ p (mod q)  p = q vô lý do q ≡ 1(mod p k ) Vậy điều giả sử là sai suy ra có vô số số nguyên tố dạng pkt + 1. Nhận xét: Đây là một trường hợp đặc biệt của định lý Dirichlet sau:

NH

“Cho a, b là hai số nguyên dương nguyên tố cùng nhau. Khi đó có vô hạn số nguyên tố có dạng an + b”. Tuy nhiên, hiện nay chưa có chứng minh sơ cấp nào cho định lý trên. Áp dụng bài toán này ta có thể dễ dàng giải quyết bài toán sau trên tạp chí Toán học và tuổi trẻ.

QU Y

Ví dụ 21 (Bình Dương 2020) Cho p là một số nguyên tố lẻ. Giả sử

(1 + x) p − 2 = 1 + a1 x + a2 x 2 + ... + a p − 2 x p − 2 .

a) Chứng minh rằng ak + (−1) k +1 ( k + 1) chia hết cho p với mọi k = 1, 2, …, p – 2.

Lời giải

DẠ

Y

a) Ta có

M

b) Khi chia a1, a2, …, ap -2 cho p ta được bao nhiêu số dư phân biệt? Vì sao?

ak + (−1)k +1 (k + 1) = C pk − 2 + (−1) k +1 (k + 1) ( p − 2)( p − 3)...( p − 1 − k ) + (−1) k +1 (k + 1) k! (−2)(−3)...(−k − 1) ≡ + (−1) k +1 (k + 1) k! k (−1) (k + 1)! ≡ + (−1) k +1 (k + 1) ≡ 0(mod p ). k!

=

Từ đó có đpcm. 21


Do đó với mọi m, n ∈ {1, 2, …, Và với mọi m, n ∈ {0, 1, 2, …,

p −3 } thì a2 n ≡ a2 m (mod p ). 2

FI CI A

k + 1 (mod p ), k = 2n ak ≡ ( −1) k ( k + 1) (mod p ) ≡   p − k − 1 (mod p ), k = 2n + 1.

L

b) Theo phần a) ta có

p −3 p −3 }thì m, n ∈ {1, 2, …, } 2 2

p −1 s ố d ư. 2

OF

Do mọi số dư của ak chia p đều là số lẻ nên chọn được đúng

Ví dụ 22. Cho hai số nguyên a, b và n là số nguyên dương. Chứng minh rằng n! là ước của số

Lời giải

Nếu n = 1, bài toán hiển nhiên đúng.

NH

Nếu n ≥ 2.

ƠN

A = b n−1a (a + b)( a + 2b)...( a + (n − 1)b).

Gọi p là một số nguyên tố bất kì, p ≤ n ta phải chứng minh vp(n!) ≤ vp(A). TH1: p | b  vp(bn - 1) = (n - 1)vp(b) ≥ n – 1

QU Y

∞  n  ∞ n ∞ n Mà v p ( n !) =   k  ≤   k  <  k = n k =1  p  k =1  2  k =1 2

 vp(n!) ≤ n – 1 ≤ vp(bn - 1) ≤ vp(A)

TH2: (p, b) = 1.

Gọi k = vp(n!) ta chứng minh B = a(a+b)(a+2b) …[a + (n - 1)b] ⋮ pk.

M

Thật vậy, do (p, b) = 1 nên tồn tại c nguyên sao cho bc ≡ 1 (mod pk) Ta có Bcn = ac(ac+bc) … [ac + (n-1)bc] ≡ ac(ac+1)(ac + 2) … (ac + (n - 1))

Do

ac(ac+1)(ac + 2) … (ac + (n - 1)) ⋮ n!

Y

nên

ac(ac+1)(ac + 2) … (ac + (n - 1)) ⋮ pk

DẠ

Suy ra Bcn ≡ 0 (mod pk)  B ≡ 0 (mod pk)

Từ đó ta có điều phải chứng minh.

22


Ví dụ 23 (NICE MO 2021). Với mỗi số nguyên tố p , kí hiệu S p = {1;2;...; p − 1} .

f : Sp → Sp thỏa mãn n. f (n). f ( f (n)) −1 là bội của p với mọi n ∈ S p . Lời giải

f

Ta chứng minh

là một đơn ánh. Thật vậy:

( f ( a )) = f ( f (b )) .

OF

Giả sử tồn tại a, b ∈ S p sao cho f (a) = f (b) . Khi đó f

FI CI A

L

Tìm tất cả các số nguyên tố p sao cho tồn tại hàm

Theo bài ra ta có

a. f ( a ). f ( f ( a ) ) − 1 ≡ b. f (b ). f ( f ( b ) ) − 1 ≡ 0 (mod p ).

ƠN

 a. f ( a ). f ( f ( a ) ) ≡ b. f (b ). f ( f ( b ) ) (mod p ). Lại có f (a) = f (b) và f ( f ( a ) ) = f ( f ( b ) ) .

( f ( a ) ) đều không chia hết cho

p (do tập giá trị của hàm

Sp ). Do đó a ≡ b (mod p) .

f

QU Y

Nhưng a, b ∈ S p nên a = b . Do đó

Nhưng tập nguồn và tập đích của hàm

f

đều là Sp có hữu hạn phần tử nên

M

{ f (1); f (2);...; f ( p − 1)} ≡ S p

{ f ( f (1) ) ; f ( f (2) ) ;...; f ( f ( p − 1) )} ≡ S

p

.

Do đó

p−1

3

1 ≡ ∏i. f (i). f ( f ( i ) ) = ( p − 1)! (mod p).

Y

i =1

Theo định lý Wilson ta có

DẠ

là một đơn ánh.

một song ánh. Từ đây ta có

f

NH

Mặt khác f (a) và f

( p −1)! ≡ −1[ p]  ( p −1)!

Vậy 1 ≡ −1 (mod p ) suy ra p = 2 . 23

3

≡ −1(mod p).

f

phải là


Thử lại thấy p = 2 là giá trị thỏa mãn.

L

a1, a2,... là dãy vô hạn các số nguyên dương. Giả sử tồn tại

số nguyên dương N đủ lớn sao cho a1 a 2 a a + + ... + n −1 + n a 2 a3 an a1

FI CI A

Ví dụ 24(IMO 2018) Cho

là số nguyên với mọi n ≥ N . Chứng minh rằng ( an ) là dãy dừng.

Lời giải

Ta có:

Sn , Sn+1 ∈ℕ nên

a1 a 2 a a + + ... + n −1 + n . a 2 a3 an a1

S n +1 − S n =

an a − an + n +1 ∈ ℤ v ới m ọi n ≥ N . a n +1 a1

ƠN

Sn =

OF

Đặt

 a1an +1 | a1an + an +1 ( an +1 − an )

Ta chứng minh bổ đề sau: nếu

NH

 a1an +1 | ( an +1 − an )( an +1 − a1 ) .

xy | ( y − x)( y − z) thì

M in {υ p ( x ) ,υ p ( z )} ≤ υ p ( y ) ≤ M ax {υ p ( x ) ,υ p ( z )}

QU Y

với mọi số nguyên tố p . Thật vậy

+) Nếu υ p ( y ) < M in {υ p ( x ) ,υ p ( z )} thì:

υ p ( ( y − x )( y − z ) ) = 2υ p ( y ) < υ p ( x ) + υ p ( y ) = υ p ( xy ) (vô lý).

M

+) Nếu υ p ( y ) >M ax {υ p ( x ) ,υ p ( z )} thì:

υ p ( ( y − x )( y − z ) ) = υ p ( x ) + υ p ( z ) < υ p ( x ) + υ p ( y ) = υ p ( xy ) (vô lý).

Bổ đề hoàn toàn được chứng minh.

Trở lại bài toán

Y

Ta có

a1an+1 | ( an+1 − an )( an+1 − a1 )

DẠ

Nên theo bổ đề trên suy ra M in {υ p ( a1 ) ,υ p ( a n )} ≤ υ p ( a n +1 ) ≤ M ax {υ p ( a1 ) ,υ p ( a n )} .

Do đó 24


gcd ( a1; an ) | gcd ( a1; an+1 )

FI CI A

gcd ( a1; an ) | a1 | lcm ( a1; an ) .

L

lcm( a1; an+1 ) | lcm( a1; an )

Từ đây ta thu được {gcd ( a1 ; a n )} là dãy số nguyên dương không giảm và bị chặn trên bởi

a1 ; {lcm ( a1 ; a n )}

là dãy số nguyên dương không tăng và bị chặn dưới bởi

đó {gcd ( a1 ; a n )} và {lcm ( a1 ; a n )} là các dãy dừng.

a1 . Do

OF

Tức là tới một lúc nào đó, ta luôn có gcd ( a1; an ) .lcm ( a1; an ) = a1.an là một hằng số, suy ra ( an ) là dãy dừng.

Đó chính là điều phải chứng minh.

2019

ƠN

Ví dụ 25 (CGMO 2019) Cho p là một số nguyên tố thỏa mãn p | 2 số ( an ) được xác định như sau

− 1 . Xét dãy

Chứng minh rằng: với mọi

thì ( an + 1) ⋮ p .

QU Y

Lời giải

n

NH

 a0 = 2, a1 = 1  .  p2 − 1 a = a + a ∀ n ≥ 1  n +1 n n −1 4 

Ta chứng minh bằng quy nạp mệnh đề sau: an+1 ≡

a0 (mod p) (đúng). 2

M

+)Với n = 0 : a1 = 1 ≡

an (mod p), với mọi n ∈ ℕ . 2

+) Giả sử mệnh đề trên đúng tới n = k ≥ 0 , tức là ak +1 ≡ Ta cần chứng minh ak +2 ≡

ak (mod p). 2

ak +1 (mod p). 2

DẠ

Y

Thật vậy

2 p2 − 1 ak +1 2ak +1 + ( p − 1) ak ak +2 = ak +1 + ak = + . 4 2 4

Vì 2 a k +1 + ( p 2 − 1 ) a k = p 2 a k + ( 2 a k +1 − a k )⋮ p . 25


4 Suy ra ak +2 ≡

≡ 0 (mod p).

ak +1 (mod p). Mệnh đề được chứng minh. 2

Bây giờ sử dụng liên tiếp mệnh đề trên ta thu được:

Ta đi chứng minh với mọi

n

an an−1 a 1 ≡ ≡ ... ≡ 1n = n (mod p). 2 4 2 2

thì ( an + 1) ⋮ p .

Thật vậy, giả sử

1 2 n −1 + 1 1 + ≡ ≡ 0 (mod p ). 2 n −1 2 n −1

ƠN

an + 1 ≡

OF

an+1 ≡

FI CI A

2ak +1 + ( p2 − 1) ak

L

mà p là một số nguyên tố lẻ nên

Khi đó

NH

2n−1 ≡ −1(mod p)  22 n−2 ≡ 1(mod p). Đặt h = ord p 2 thì h | 2 n − 2 và h | 2019 . Vì 2019 là số lẻ nên h | n − 1 .

QU Y

Nhưng khi đó thì 2 n −1 ≡ 1 (mod p ), và do đó 1 ≡ −1(mod p) ⇔ p = 2 (vô lý). Vậy ( an + 1) ⋮ p với mọi n.

Ví dụ 26. Cho m, n nguyên dương, n > 1. Chứng minh rằng {1n, 2n, …, mn} là hệ thặng dư đầy đủ mod m khi và chỉ khi m là số square-free và (n, j(m)) = 1.

M

Lời giải

Phần thuận: Giả sử {1n, 2n, …, mn} là hệ thặng dư đầy đủ mod m. t

Xét m = p a ∏ pia (với p, p1, …, pt là các số nguyên tố, a nguyên dương). i

i =1

DẠ

Y

+ Nếu a ≥ 2, khi đó ta xét t  x ≡ y (mod piai ) a t ∏   2   i =1  x ≡ y (mod p piai ) (1)  ∏ a i =1     x ≡ y (mod p  2  ) n

n

 x ≡ y (mod p

a n  2

t

ai i

∏p i =1

26

)


a

L

Do n ≥ 2 Þ n   ≥ a  x n ≡ y n (mod m) 2

FI CI A

Ta có t

p a ∏ piai

m p

a 2  

t

i =1

= ai i

∏p

p

a  2  

i =1

t

=p

a a−  2

≥ p (do a ≥ 2)

ai i

∏p i =1

Nên từ (1) suy ra tồn tại x ≠ y thuộc (1, 2, …, m) thoả mãn xn º yn (mod m) (vô lý).

OF

Vậy m là số square – free. t

Suy ra m = ∏ pi i =1

+) Nếu (n, j(m)) = d > 1 Þ tồn tại số nguyên tố q | d Þ q | pi – 1, với i nào đó.

x pi −1 − 1 ≡ 0(mod pi ) là phương trình đa thức bậc pi – q – 1 nên xq −1

có tối đa (pi – q - 1) nghiệm.

NH

Thật vậy phương trình

ƠN

Bổ đề: Phương trình x3 º 1 (mod pi) có ít nhất q nghiệm.

Mà x p −1 − 1 ≡ 0(mod pi ) có đúng pi – 1 nghiệm trên ℤ p nên phương trình xq – 1 º 0 i

(mod pi) có ít nhất q nghiệm. (Bổ đề được chứng minh)

QU Y

Gọi a, b là 2 trong số các nghiệm đó.

Theo định lý thặng dư thặng dư Trung Hoa, tồn tại các số tự nhiên n1, n2 sao cho n1 º b (mod pj), j ≠ i; n1 º a (mod pi); n2 º b (mod pi). Dễ thấy có thể chọn n1, n2 Î {1, 2, …, m} và n1 ≠ n2. Hiển nhiên ta có n1q ≡ n2q (mod m)  n1n ≡ n2n (mod m) (vô lý)

M

Vậy phần thuận được chứng minh.

Ví dụ 28 (GGTH) Đặt S(n, k) =

n −1

i

k

với n, k nguyên dương, n lẻ. Chứng minh rằng

i =1

với n = p là số nguyên tố lẻ thì a) S(p,k) º -1 mod p Û k chia hết cho p – 1.

Y

b) S(p,k) º 0 mod p Û k không chia hết cho p – 1.

DẠ

Lời giải Nếu k ⋮ p - 1 thì k

i = (p

k p −1 p −1

)

≡ 1(mod p ), ∀i = 1, p − 1.

27


Do đó p −1

L

S ( p, k ) ≡ 1 = p − 1 ≡ −1(mod p).

Nếu k ≠ p – 1 thì k = (p - 1)q + r, 1 ≤ r ≤ p – 2, q Î ℕ .

FI CI A

i =1

 i k ≡ i r (mod p )  S ( p , k ) ≡ S ( p , r )(mod p ).

Ta có phương trình xr º 1 (mod p) có không quá r nghiệm.

Mà r Î {1, 2, …, p - 2} nên tồn tại a Î {1, 2, …, p - 1} sao cho a r ≡ 1(mod p ).

Do đó p −1

p −1

p −1

i =1

i =1

i =1

OF

Vì (a, p) = 1 nên {ia}ip=−11 là hệ thặng dư thu gọn (mod p).

S ( p, r ) =  i r ≡  (ia ) r ≡ a r  i r (mod p) r

ƠN

 ( a − 1) S ( p, r ) ≡ 0(mod p ).

Do (ar – 1, p) = 1 Þ S(p, r) º 0 (mod p).

NH

Nhận xét. Ta có thể gọi a là một căn nguyên thuỷ mod p thì không cần sử dụng phương trình đồng dư, bạn đọc có thể xem ở lời giải bài sau (Ví dụ 29). Bài toán trên thực chất là một cách phát biểu khác của bai toán Hunggary – Israel năm 2009, bạn đọc có thể tìm thấy một lời giải khác trên mathlinks.ro! n −1

i

QU Y

Ví dụ 29 (Mạc Đăng Nghị). Đặt S(n, k) =

k

với n, k nguyên dương, n lẻ. Chứng

i =1

minh rằng với n = pm, m nguyên dương thì a) S(pm, k) º 0 mod pm nếu k không chia hết cho p – 1; b) S(pm, k) º -pm – 1 mod pm nếu k chia hết cho p – 1.

Lời giải

M

(Bài này là trường hợp tổng quát của bài trên)

a) Xét k ⋮ p – 1.

Với p lẻ bất kì. Gọi g là một căn nguyên thuỷ của pm. Khi đó {g, 2g, …(pm - 1)g} và {1,2,…, pm - 1} là hai hệ thặng dư giống nhau (mod pm).

DẠ

Y

Khi đó ta có

gk + (2g)k + … + ((pm - 1)g)k º 1k + 2k + … + (pm - 1)k

Þ (gk - 1)S(pm, k) º 0 (mod pm).

Do j(pm) = pm – 1(p - 1) nên j(pm) ⋮ p – 1. Lại do g là căn nguyên thuỷ (mod p) nên 28


gk – 1 º 0 (mod pm) Û k ⋮ j(pm).

Vậy khi đó S(pm, k) º 0 (mod pm). b) Xét k ⋮ p – 1. Ta chứng minh bằng quy nạp. Nếu m = 1, Theo định lý Fermat nhỏ, ta có

FI CI A

gk – 1 ⋮ pm và (gk – 1, p) = 1

L

Do đo nếu k ⋮ p – 1 thì k ⋮ j(pm) nên

OF

S(p, k) = 1k + 2k + … +(p - 1)k º 1 + 1 + … + 1 º - 1 (mod p). Giả sử S(pn, k) º - pn – 1 (mod pn). Ta có

ƠN

S ( p n +1 , k ) = 1k + 2k + ... + ( p n +1 − 1) k

= 1k + 2k + ... + ( p n − 1)k + ( p n )k + [(p n +1)k + ( p n + 2) k + ... + ( p n + p n − 1) k ] + ... + [(( p − 1) p n + 1) k + (( p − 1) p n + 2) k + ... + (( p − 1) p n + p n − 1) k .

NH

Do p – 1 | k Þ k ≥ 2 Þ kn ≥ n + 1 Suy ra p −1

S ( p n +1 , k ) ≡   (ip n + 1) k + (ip n + 2)k + ...(ip n + p n − 1)k  i =0

p −1

QU Y

≡   (1k + 2k + ... + ( p n − 1)k ) + kip n (1k −1 + 2k −1 + ... + ( p n − 1) k −1 )  (mod p n +1 ) i =0

≡ pS ( p n , k ) + k (1 + 2 + ... + ( p − 1)) p n S ( p n , k − 1)(mod p n +1 ) ≡ pS ( p n , k ) + k (

Do

p ( p − 1) n ) p S ( p n , k − 1)(mod p n +1 ) ≡ pS ( p n , k )(mod p n +1 ). 2

Suy ra

M

S(pn, k) º - pn – 1 (mod pn ) Þ S(pn, k) ≡ -pn – 1 + m.pn (mod p).

S ( p n +1 , k ) ≡ pS ( p n , k )(mod p n +1 ) ≡ p (− p n −1 + m. p n ) ≡ − p n + m. p n +1 ≡ − p n (mod p n +1 ).

Từ đó có điều phải chứng minh. n(n+1)(n+2)(n+3)

DẠ

Y

Ví dụ 30 (Hà Tĩnh TST 2014) Tìm tất cả các số nguyên dương n sao cho tích có đúng 3 ước số nguyên tố.

Lời giải

29


Vậy ta xét trường hợp A có đúng 3 ước số là 2; 3; p (p ≥ 5).

+) n lẻ (n ≥ 3).

FI CI A

Vì A là tích của 4 số nguyên dương liên tiếp nên A luôn có ước số là 2; 3.

L

Với n = 1, đặt A = n(n +1)(n +2)(n+3) = 1.2.3.4= 23.3 chỉ có 2 ước số nguyên tố, do đó ta chỉ xét với n ≥ 2.

Khi đó n và n + 2 là 2 số lẻ liên tiếp, nguyên tố cùng nhau nên suy ra n, n + 2 là hai luỹ thừa 3a và pb. Vì n + 1 chẵn và nguyên tố cùng nhau với n và n + 2 nên n + 1 = 2k , k ≥ 2.

OF

Từ đó suy ra

A = (2k - 1)2k(2k +1)(2k + 2) = 2k+1(2k - 1)(2k +1)(2k-1 +1) Vì k ≥ 2 nên 3 thừa số sau ở (1) đều lẻ.

ƠN

Mặt khác

( (2k +1 , 2k - 1 ) = (2k +1, 2k +1 -2) = (2k +1, -2) = 1 và

NH

(2k +1, 2k-1 +1) = (2(2k-1+1) -1, 2k-1 +1) = (-1, 2k-1 +1) = 1. Suy ra:

Khi đó

QU Y

2k − 1 = 3a  +) Hoặc 2k + 1 = p b , với a, b, c nguyên dương. 2k −1 + 1 = 3c 

3a = 2k – 1 = 2(2k – 1 + 1) – 3 = 2.3c - 3.

Từ đó ta có

M

3a – 1 = 2.3c – 1 - 1

Đẳng thức xảy ra khi a – 1 = c – 1 = 0 Þ a = c = 1 Þ n = 2k – 1 = 3a = 3.

DẠ

Y

 2k − 1 = p a  +) Hoặc 2k + 1 = 3b với a, b, c nguyên dương. 2k −1 + 1 = 3c 

Khi đó pa = 2k – 1 = 2.pc – 3. 30

(1).


Điều này không xẩy ra vì a, c ≥ 1 mà 3 không chia hết cho p.

L

Vậy với n lẻ chỉ có n = 3 thoả mãn.

FI CI A

+) n chẵn ( n ≥ 2). Khi đó (n + 1), (n + 3) là hai số lẻ liên tiếp nguyên tố cùng nhau suy ra (n + 1), (n + 3) là hai luỹ thừa 3a và pb.

Vì n + 2 chẵn, nguyên tố với n +1 và n + 3 nên n + 2 = 2k với k ≥ 2. Khi đó

(2)

OF

A = (2k - 2)(2k -1)2k(2k + 1) = 2k+1( 2k-1-1)(2k -1)(2k +1)

+) Nếu k = 2 thì 2k-1 - 1= 2 - 1 = 1 và n = 2k - 2 = 22 - 2 = 2, thì A = 2.3.4.5 = 23.3.5 có đúng 3 ước số nguyên tố.

ƠN

+) Nếu k ≥ 3 thì 3 thừa số sau ở (2) đều là số lẻ, mặt khác ( 2k -1, 2k +1) = 1, ( 2k -1, 2k-1 -1 ) = (2(2k-1-1) +1, 2k-1- 1) = 1. Khi đó chỉ có thể xảy ra một trong hai trường hợp sau:

NH

 2k −1 − 1 = 3a  +) Hoặc  2k + 1 = p b với a, b, c nguyên dương.  2k −1 + 1 = 2c 

Suy ra

QU Y

3c = 2k + 1 = 2.3a + 3 Û 3c – 1 = 2.3a – 1 + 1

Û c – 1 = 2; a – 1 = 0 Û c = 2; a = 1

Khi đó n = 6

Suy ra

M

2k −1 − 1 = p a  +) Hoặc 2k + 1 = 3b với a, b, c nguyên dương. 2k −1 + 1 = 2c 

pc = 2k + 1 = pa + 3.

Điều này không xẩy ra vì a, c nguyên dương, p > 3.

Y

và p nguyên tố, p > 3.

DẠ

Vậy với n chẵn thì chỉ có n = 2, n = 6 thoả mãn.

Vậy n = 2, n = 3, n = 6.

Ví dụ 31 (Đề KT đội tuyển Hà Tĩnh 2015). Tìm tất cả các bộ ba các số nguyên dương (a, m, n) sao cho 31


( a + 1)

n

m

L

a +1

∈ ℕ*

FI CI A

Lời giải

Từ tính chất duy nhất trong phân tích số nguyên dương thành thừa số nguyên tố ta có: nếu vk chia hết cho u, thì (glc(u,v))k chia hết cho u. (1) Dễ thấy các bộ (1, m, n ) và (a, 1, n) với mọi số nguyên dương a, m, n là các nghiệm của bài toán.

Bổ đề: Nếu a > 1 và (a +1)n chia hết cho

am +1 thì m lẻ.

OF

Chứng minh bổ đề

Nếu m chẵn thì am + 1 ≡ 2 (mod (a+1)), nên (am + 1, a +1) bằng 1 hoặc 2. Từ (1) suy ra

ƠN

2n ⋮ am + 1 Þ am +1 = 2s.

Ta có s ≥ 2 vì a > 1. Khi đó am = 2s - 1 ≡ - 1(mod 4), điều này không xẩy ra vì am là một số chính phương.

NH

Như vậy a > 1 và m là số nguyên dương lẻ lớn hơn 1 và hiển nhiên n > 1. Gọi p là số nguyên tố lẻ sao cho m = pr, b = ar.

Vì r lẻ, b + 1 chia hết cho a + 1, nên (b+1)n chia hết cho bp +1 = am + 1. Từ đó suy ra

QU Y

(b +1)n -1 ⋮ B = (bp + 1)/(b + 1).

Theo (1) thì (B, b+1)n-1 chia hết cho b. Vì p lẻ và từ công thức nhị thức suy ra b p + 1 (b + 1 − 1) p + 1 = ≡ p (mod (b + 1)). b +1 b +1

M

Vậy số nguyên tố p chia hết cho (B, b +1) suy ra (B, b+1) = p.

Từ đó pn-1 chia hết cho B, suy ra B là luỹ thừa của p. Ta có bp +1 chia hết cho p do đó (b+ 1)n chia hết cho p, cho nên b + 1 ⋮ p Þ b = kp -1.

DẠ

Y

Áp dụng công thức nhị thức ta có: 2 2 2 2 2 3 bp +1 = (kp - 1)p +1 = ((kp)p -…- C p (kp) + kp − 1) ≡ kp (mod k p )

B=

b p +1 b p +1 = ≡ p (mod kp 2 ) b +1 kp

Điều này chứng tỏ b chia hết cho p nhưng không chia hết cho p2 . 32


Vì B là một luỹ thừa của p, suy ra B = p.

FI CI A

b p +1 = b p −1 − b p −2 + ... − b + 1 > b p −1 − b p −2 = (b − 1)b p −2 ≥ 2 p −2 > p. b +1

Từ đó ta có:

p = 3, B = p Þ b2 - b +1 = 3 Þ ar = b = 2 Þ a = 2 và m = p = 3. Vậy (2, 3, n) là nghiệm của bài toán với n ≥ 2

Vậy nghiệm của bài toán là: (1, m, n); (a, 1, n) với a, m, n ∈ N * ; (2, 3, n)

DẠ

Y

M

QU Y

NH

ƠN

OF

với n ≥ 2, n Î ℕ .

33

L

Trường hợp nếu p ≥ 5 thì


L

Chương 2. Thặng dư bậc hai

FI CI A

Phần này, bản thân tôi cũng đã báo cáo trong chương trình bồi dưỡng giáo viên chuyên Toán, năm đầu tiên do Viện NCCC về Toán tổ chức. Qua đó, cũng nhận được khá nhiều góp ý của các bạn đồng nghiệp và chuyên đề cũng đã được chỉnh sửa nhiều cho phù hợp với thực tiễn giảng dạy học sinh chuyên toán hiện nay.

2.1. Lí thuyết

OF

2.1.1 Các định nghĩa Định nghĩa 1

Cho số nguyên tố p. Số nguyên a được gọi là số chính phương (mod p) nếu tồn tại số nguyên x sao cho x2 ≡ a ( mod p).

ƠN

Nhận xét:

+) Mọi số chính phương đều là số chính phương (mod p)

NH

+) a ≡ 0 (mod p) thì a2 ≡ a (mod p) nên mọi a ≡ 0 (mod p) đều là số chính phương (mod p). Do đó, từ đây về sau, ta chỉ xét số nguyên a sao cho (a, p) = 1. +) Mọi số nguyên lẻ đều là số chính phương (mod 2)

Định nghĩa 2 ( Kí hiệu Legendre)

QU Y

Cho p là số nguyên tố lẻ.

a a   = 0 , nếu a ≡ 0 (mod p).   = 1 nếu a là số chính phương mod p  p  p a   = -1 nếu a không là số chính phương mod p.  p

M

Kí hiệu trên gọi là kí hiệu Legendre.

2.1.2 Các định lí

Định lý 1

DẠ

Y

Cho p là một số nguyên tố lẻ. Khi đó p −1 a 2 = 1 ⇔ a ≡ 1(mod p )    p

(1)

p −1 a 2 = − 1 ⇔ a ≡ −1(mod p )    p

(2)

34


Chứng minh

L

Giả sử a là số chính phương mod p, khi đó, tồn tại số tự nhiên x sao cho

FI CI A

x2 ≡ a (mod p) Do (a, p) = 1 nên (x, p) = 1. Theo định lí Fermat ta có 1≡ x

p −1

≡a

p −1 2

(mod p )

OF

Ngược lại, nếu có (1) thì với mỗi k ∈ {1, 2, …, p - 1} có duy nhất một số k ' ∈ {1,2,...,p-1} sao cho k.k’ ≡ a (mod p) Nếu tồn tại k = k’ thì k.k’ = k2 ≡ a (mod p)  a là số chính phương (mod p).

≡ a (mod p).  (p - 1)! ≡ a

p −1 2

≡ 1(mod p )

Mặt khác, theo định lí Wilson, ta có

Từ đó suy ra

NH

(p - 1)! ≡ −1 (mod p)

k −1 tập con {k, k’} rời nhau sao cho k.k’ 2

ƠN

Trái lại, tập {1, 2, …, p - 1} được chia thành

1 ≡ −1 (mod p)  ‫ = ݌‬2 (vô lí vì p lẻ)

QU Y

Vậy (1) được chứng minh xong.

Theo định lí Fermat, ta có ap – 1 ≡ 1 (mod p)

 p2−1  a ≡ 1(mod p)   p −1  a 2 ≡ −1(mod p)

M

Do đó, (2) được chứng minh.

Định lí 2 (Bổ đề Gauss) Cho p là số nguyên tố lẻ, a là số nguyên, (a, p) = 1. Xét tập

Y

{ka| k = 1, 2, …,

p −1 }. 2

DẠ

Gọi rk ≡ ka (mod p), 1 ≤ rk ≤ p.

p Gọi n là số các số rk thuộc khoảng ( ; p ). Khi đó a 2

35

p −1 2

≡ ( −1) n (mod p ) .


p 2

(n chính là số bội số của a trong khoảng ( ; p ))

L

Chứng minh

FI CI A

Ta có ka ≡ rk (mod p)

Cho k chạy từ 1 đến (p - 1)/2 rồi nhân các đẳng thức đó lại ta được: p −1

2 p − 1 p2−1 ! a ≡ ∏ rk (mod p ) 2 k =1

OF

p p   Gọi A = rk | rk >  ; B = rk | rk <  2 2  

p −1 2

∏r = ∏r ∏r k =1

k

rk ∈A

k

rk ∈B

Ta có với mọi rk ∈ A thì p – rk ≤

≡ (−1)n ∏ ( p − rk )∏ rk (mod p) rk ∈A

p −1 . 2

rk ∈B

NH

k

ƠN

Khi đó

Mặt khác Nếu ri ∈ A, rj ∈ B thì p – ri ≠ rj vì nếu p – ri = rj thì ri + rj = p

 a(i + j) ≡ 0 (mod p)  i + j ≡ 0 (mod p).

Từ đó suy ra

p −1 !a 2

p −1 2

p −1 2

≡ ∏ rk ≡ ( −1) n k =1

p −1 !, p ) = 1 nên từ (*) suy ra a 2

p −1 ! 2

p −1 2

(*)

≡ (−1)n (mod p )

đpcm.

p −1 ) nên 1 < i + j < p . 2

M

Mà (

QU Y

Điều này không thể vì i và j thuộc (1;

Từ đó ta có một số hệ quả quan trọng sau. p 2

Hệ quả 1. Cho p là số nguyên tố lẻ. Gọi n là số các số chẵn thuộc ( ; p ) thì ta

DẠ

Y

2

p −1 2

≡ ( −1) n (mod p ) .

Chứng minh Từ định lí 2, cho a = 2 ta có điều phải chứng minh. 36


Hệ quả 2. Cho p là số nguyên tố lẻ. Khi đó

Chứng minh (Hiển nhiên)

Hệ quả 3. Cho p là số nguyên tố lẻ. Khi đó 2 ( ) = 1 ⇔ p ≡ ± 1 (mod 8) p 2

OF

p −1 2 và   = ( −1) 8  p

FI CI A

L

p −1 −1 ( ) = ( −1) 2 p

Chứng minh

p 2

ƠN

Giả sử p = 4k + 1, tập các số chẵn trong khoảng (0; p) là {2i | 1 ≤ i ≤ 2k} Khi đó, số các số chẵn trong khoảng ( ; p ) là n = k.

p ; p ) là n = k. 2

Do đó, theo hệ quả 1, ta có

≡ ( −1) n (mod p ) ≡ (-1)k (mod p)

QU Y

2

p −1 2

NH

Tương tự như vậy, nếu p = 4k – 1 thì ta cũng tính được số các số chẵn trong khoảng (

2 là số chính phương mod p ⇔ 1 ≡ 2

p −1 2

≡ ( −1) k (mod p )

⇔ k chẵn ⇔ p ≡ ± 1 (mod p).

đpcm.

M

2 không là số chính phương mod p ⇔ k lẻ ⇔ p ≡ ± 3 (mod p)

p 2

Hệ quả 4. Cho p là số nguyên tố lẻ. Gọi n là số các bội của 3 trong khoảng ( ; p

). Khi đó

3

p −1 2

≡ ( −1) n (mod p )

Y

Chứng minh

DẠ

Được suy ra hiển nhiên từ định lí, trong trường hợp a = 3.

37


L

Hệ quả 5. Cho p là số nguyên tố có dạng 6k ± 1. Khi đó, 3 là số chính phương mod p khi và chỉ khi p ≡ ± 1 (mod 12).

FI CI A

Chứng minh Giả sử p = 6k + 1. Tập hợp các số là bội của 3 trong khoảng (0; p) là {3i | 1 ≤ i ≤ 2k} Mà 3i >

p p 1 ⇔ i > = k + ⇔ i ≥ k +1 2 6 6 p 2

OF

Do đó, số các số là bội của 3 trong khoảng ( ; p ) là n = k. Tương tự khi p = 6k – 1, ta cũng chứng minh được n = k.

ƠN

Vậy trong cả hai trường hợp ta đều chứng minh được số các số là bội của 3 trong khoảng p ( ; p ) là n = k. 2 Vì vậy, theo hệ quả 4, ta có

≡ ( −1) n (mod p ) ≡ (-1)k (mod p)

NH

3

p −1 2

3 p

 ( ) = 1 ⇔ (-1)k ≡ 1 (mod p) ⇔ k chẵn

QU Y

⇔ p ≡ ± 1 (mod 12). đpcm.

Định lí 3

Cho p là số nguyên tố. a ≡ b (mod p), (a, p) = (b, p) = 1. Khi đó

a b ( ) = ( ). p p

M

Chứng minh

Do a ≡ b (mod p)  a

p −1 2

≡b

p −1 2

a b (mod p)  ( ) = ( ) p p

đpcm.

Y

Định lí 4

Cho p là một số nguyên tố. a1, a2, …, an là các số nguyên không chia hết cho p.

DẠ

Khi đó

(

a1a2 ...an a a a ) = ( 1 )( 2 )...( n ) p p p p 38


Chứng minh

FI CI A

L

(Phần chứng minh khá đơn giản, xin nhường cho bạn đọc)

Định lí 5

Cho p là số nguyên tố lẻ, a là số nguyên không chia hết cho p. Khi đó a

≡ ( −1) s (mod p )

 ka 

 p  k =1

OF

Với s =

p −1 2

p −1 2

Chứng minh p 2

a

p −1 2

ƠN

Gọi n là số các bội của a trong khoảng ( ; p ), theo định lí 2, ta có ≡ (−1)n (mod p )

Vì vậy, ta chỉ cần chứng minh s – n là số chẵn.

NH

(Phần này khá đơn giản, xin nhường lại cho bạn đọc).

Định lí 6 (Luật tương hỗ Gauss)

Cho p, q là hai số nguyên tố lẻ phân biệt.

QU Y

Khi đó:

a) Nếu có ít nhất một trong hai số có dạng 4k + 1 thì p q ( )=( ). q p

b) Nếu cả hai số có dạng 4k + 3 thì

M

p q ( ) = −( ). q p

Chứng minh

Y

a) Đặt

p −1 2

q −1

2   iq  ip  s1 =   ; s2 =    i =1  p  i =1  q 

DẠ

Khi đó s1 + s2 = (

p −1 q −1 )( ) 2 2

39


p −1 q −1 )( ) là số chẵn. 2 2

Nên s1 và s2 có cùng tính chẵn lẻ. Theo định lí 5, ta có q

p −1 2

≡ (−1) s1 (mod p )

q

Từ đó dễ dàng suy ra điều phải chứng minh.

ƠN

b) Chứng minh tương tự.

OF

p 2 ≡ (−1) s2 (mod q )

FI CI A

Do p hoăc q có dạng 4k + 1 nên (

L

(tính chất này đã được chứng minh trong tài liệu “Một số bài toán về phần nguyên ” của cùng tác giả).

2.2. Các ví dụ 2.2.1 Các ví dụ cơ bản

NH

Phần này, chúng tôi cố gắng lựa chọn các ví dụ đơn giản nhất về thặng dư bậc hai. Giúp học sinh có hình dung cơ bản về định hướng sử dụng thặng dư bậc hai trong các bài toán số học.

Ví dụ 1. Chứng minh rằng mọi ước nguyên tố của số m = 2n – 1, (n lẻ) đều có dạng

QU Y

p = 8k ± 1.

Lời giải Ta có

p | 2n – 1  2n ≡ 1 (mod p) ⇔ (2m)2.2 ≡ 1 (mod p).

M

2 ⇔ (2m + 1)2 ≡ 2 (mod p) ⇔   = 1 ⇔ p = 8k ± 1.  p

Từ đó có điều phải chứng minh.

Ví dụ 2 (JBMO 2007). Cho p là số nguyên tố. Chứng minh rằng số 7p + 3p – 4

không là số chính phương.

Y

Lời giải

DẠ

TH1: p = 2 ta có 7p + 3p – 4 = 19 không là số chính phương.

TH2: p lẻ. Giả sử 7p + 3p – 4 = x2 với x nguyên dương.

Ta có 3p ≡ 3 (mod p)  7p + 3p – 4 ≡ -1 (mod p). 40


L

 −1  Suy ra   = 1  p ≡ 1 (mod 4).  p

Từ đó ta có điều phải chứng minh.

FI CI A

Xét (mod 4), ta có 7p + 3p – 4 ≡ 7 + (-1)p ≡ 2 (mod 4)  x2 ≡ 2 (mod 4), vô lí vì một số chính phương chia 4 không thể dư 2.

Ví dụ 3. Cho p là số nguyên tố lẻ. Khi nào p – 2 là số chính phương (mod p)? Lời giải Ta có 2

OF

p −1 p −1 ( p −1)( p + 5)  p − 2  −2  −1   2  8 8 2 = ( ) = = ( − 1) ( − 1) = ( − 1)      p  p   p  p 

Do đó p – 2 là số chính phương (mod p) ⇔ p ≡ 1 hoặc 3 (mod 8).

ƠN

Ví dụ 4. Cho p = 8n – 1 và q = 4n – 1 là các số nguyên tố. Chứng minh rằng -2 là một căn nguyên thủy (mod p). Ta có

NH

Lời giải

(−2)q = (−2)

 −2  ≡   (mod p)  p

QU Y

 −2  Mà p = 8n – 1 nên   = -1.  p

p −1 2

Mặt khác p – 1 = 2q  (-2)2q ≡ (-2)p -1 ≡ 1 (mod p). Nếu – 2 không là căn nguyên thủy (mod p) thì tồn tại r sao cho 0 < r < p – 1 và (-2)r ≡ 1 (mod p)  r | 2q  r = 2.

M

Mà 1 ≡ (-2)2 ≡ 4 (mod p)  p = 3 (không thỏa mãn). Vậy có điều phải chứng minh.

Ví dụ 5. Cho p là số nguyên tố có dạng 8k + 5 hoặc 8k + 7. x, y là hai số nguyên thỏa mãn x2 + 2y2 chia hết cho p. Chứng minh rằng x ⋮ p và y ⋮ p.

Y

Lời giải

DẠ

Giả sử ngược lại x ⋮ p  2y2 ⋮ p  y ⋮ p.

Mà x2 + 2y2 ⋮ p  x2 ≡ -2y2 (mod p) 41


1= (

−2 y 2 −2 −1 2 )( ) = ( ) = ( )( ) . p p p p p

p −1  −1 2 ( ) = ( − 1) =1  p  Nếu p = 8k + 5   p 2 −1  2 8 = −1. ( p ) = (−1) 

(1)

OF

Thay vào (1) được 1 = -1 (vô lí).

L

x2 −2 y 2 )=( ) p p

FI CI A

(

ƠN

p −1  −1 2 ( ) = ( − 1) = −1   p Nếu p = 8k + 1   cũng không thoả mãn (1) p 2 −1  2 8 =1 ( p ) = (−1) 

Vậy điều giả sử là sai Từ đó suy ra đpcm.

NH

Ví dụ 6. Cho p là số nguyên tố lẻ. Chứng minh rằng nếu tồn tại các số nguyên x, y sao cho p = x2 + 2y2

QU Y

thì p ≡ 1 (mod 8) hoặc p ≡ 3 (mod 8).

Lời giải

Dễ thấy cả x và y đều không chia hết cho p. Ta có x2 ≡ -2y2 (mod p)

M

Do đó

 x 2   −2 y 2  −2 1=   =  =( ) p  p  p 

Suy ra p ≡ 1 hoặc 3 (mod 8).

Ví dụ 7. Cho p là số nguyên tố lẻ và p ≡ 1 hoặc 3 (mod 8). Chứng minh rằng luôn tồn tại các số nguyên x, y sao cho p = x2 + 2y2.

Y

Lời giải

DẠ

 −2  Vì p ≡ 1 hoặc 3 (mod 8) nên   = 1.  p

Khi đó tồn tại r nguyên dương sao cho r2 ≡ -2 (mod p). 42


Đặt f(u, v) = u + rv và k =  p  .

Xét tất cả các cặp (u, v) với 0 ≤ u ≤ k và 0 ≤ v ≤ k Có tất cả (k + 1)2 cặp phân biệt.

FI CI A

L

Khi đó k <  p  < k + 1 do p là số nguyên tố.

Mà (k + 1)2 > p nên theo nguyên lý Dirchlet tồn tại (u1, v1) ≠ (u2, v2) sao cho f(u1, v1) ≡ f(u2, v2) (mod p).

⇔ u1 + rv1 ≡ u2 + rv2 (mod p) ⇔ (u1 – u2) ≡ - r(v1 – v2) (mod p).

OF

Hay tồn tại a, b nguyên không đồng thời bằng 0 sao cho

a ≡ -rb (mod p)  a2 ≡ r2b2 (mod p)  a2 ≡ -2b2 (mod p). ⇔ a2 + 2b2 ≡ 0 (mod p).

ƠN

Mà 0 ≤ |u1 - u2| ≤ k và 0 ≤ |v1- v2 | ≤ k nên a2 + 2b2 ≤ 3k2 < 3p. Từ đó suy ra a2 + 2b2 = p hoặc a2 + 2b2 = 2p.

Nếu a2 + 2b2 = 2p thì a phải là số chẵn hay a = 2c suy ra b2 + 2c2 = p.

NH

Từ đó suy ra điều phải chứng minh.

Ví dụ 8. Chứng minh rằng mọi ước nguyên tố lẻ, lớn hơn 3 của số n = x2 + 3y2, x, y nguyên dương ( x ⋮ p, y ⋮ p) đều có dạng p = 6k + 1.

QU Y

Lời giải

 x2 

−3 y 2

−1 3

Ta có x2 ≡ - 3y2 (mod p)  1 =   = ( ) = ( )( ) p p p  p  −1  3 Nếu p = 12k + 5 thì   = 1,   = −1 nên trường hợp này không thỏa mãn.  p  p

M

 −1  3 Nếu p = 12 k + 11 thì   = −1,   = 1 nên trường hợp này không thỏa mãn.  p  p

 −1  3 Nếu p = 12k + 1 thì   = 1,   = 1 (thỏa mãn).  p  p

Y

 −1  3 Nếu p = 12k + 7 thì   = −1,   = −1 (thỏa mãn).  p  p

DẠ

Như vậy cả hai trường hợp sau đều dẫn đến p = 6k + 1. Từ đó có điều phải chứng minh.

Ví dụ 9. Cho p > 3, p là số nguyên tố thỏa mãn p = x2 + 3y2 với x, y nguyên. Chứng minh rằng p ≡ 1 (mod 6). 43


Chứng minh tương tự ví dụ trên.

Lời giải Giả sử phương trình trên có nghiệm y.

FI CI A

L

Ví dụ 10. Cho p, q là các số nguyên tố lẻ. a không là số chính phương (mod p). Chứng minh rằng phương trình x2 ≡ a (mod pq) vô nghiệm.

Khi đó y2 ≡ a (mod pq)  pq | y2 – a  p | y2 – a  y2 ≡ a (mod p) Từ đó suy ra a là số chính phương (mod p) (trái với giả thiết). Vậy có điều phải chứng minh. p = 3.2n + 1 trừ trường hợp p = 13.

ƠN

Lời giải

OF

Ví dụ 11. Chứng minh rằng 2 không là căn nguyên thủy của một số nguyên tố dạng

TH1 : p = 7 thì 23 = 8 ≡ 1 (mod 7) nên 2 không là căn nguyên thủy (mod 7). TH2: p = 13 thì dễ dàng kiểm tra được 2 là một căn nguyên thủy của 13.

Vậy có điều phải chứng minh.

NH

p −1 2 TH3: n ≥ 3 thì p ≡ 1 (mod 8)    = 1  2 2 ≡ 1(mod p)  2 không là căn nguyên  p thủy (mod p).

QU Y

Ví dụ 11. Cho p là số nguyên tố lẻ, a ∈ ℤ sao cho (a, p) = 1. Chứng minh rằng nếu a là số chính phương (mod p) thì a cũng là số chính phương (mod pn), với mọi n nguyên dương. Lời giải

M

Giả sử rằng b2 ≡ a (mod p), chúng ta cần chứng minh tồn tại số x nguyên sao cho x2 ≡ a (mod pn)

(*)

với mọi n nguyên dương. Ta sẽ chứng minh bằng quy nạp.

Y

Với n = 1, hiển nhiên (*) đúng. Giả sử tồn tại c sao cho c2 ≡ a (mod pn), với n ≥ 1, ta cần chứng minh tồn tại x sao cho x2 ≡ a (mod pn + 1).

DẠ

Từ c2 ≡ a (mod pn) suy ra c2 = a + pnk, k nguyên. Xét tất cả các số có dạng x = c + pni với i nguyên, ta có x2 = c2 + 2cpni + p2ni2 44


= a +pn(2ci + k) + p2ni2

L

≡ a + pn(2ci + k) (mod p)

FI CI A

Ta sẽ chứng minh tồn tại i sao cho 2ci + k ≡ 0 (mod p)

Thật vậy, do p nguyên tố lẻ, c ⋮ p nên (2c, p) = 1 theo định lí Bezout tồn tại các số nguyên y, z sao cho 2cy + pz = 1  2cy ≡ 1 (mod p)

 -2cyk ≡ - k (mod p)  -2cyk + k ≡ 0 (mod p)

OF

Từ đó chọn i = -yk ta được 2ci + k ≡ 0 (mod p)  x2 ≡ a (mod pn + 1) Vậy có điều phải chứng minh.

Chú ý: Nếu a là số chính phương (mod pn) thì p cũng là số chính phương (mod p)

Lời giải k

ƠN

Ví dụ 12. Cho m là số nguyên dương lẻ, a nguyên dương sao cho (a, m) = 1. Chứng minh rằng a là số chính phương (mod m) khi và chỉ khi a là số chính phương (mod p) với mọi p là ước nguyên tố của m.

NH

Giả sử phân tích chính tắc của m là m = ∏ piα . i

i =1

Theo định lí Thặng dư Trung Hoa ta có phương trình x2 ≡ a (mod m) có nghiệm khi và chỉ khi hệ

QU Y

 x 2 ≡ a (mod p1α1 )  2 α  x ≡ a (mod p2 2 )  ...  x 2 ≡ a (mod pα k ). k 

M

Mà theo ví dụ trên, phương trình x2 ≡ a (mod pn) có nghiệm khi và chỉ khi phương trình x2 ≡ a (mod p) có nghiệm. Từ đó có điều phải chứng minh.

Ví dụ 13. Cho p là số nguyên tố lẻ, a là số nguyên sao cho (a, p) = 1. Chứng minh rằng phương trình x2 + py + a = 0

có nghiệm nguyên (x, y) khi và chỉ khi –a là số chính phương (mod p).

Y

Lời giải

DẠ

Giả sử phương trình x2 + py + a = 0 có nghiệm nguyên.  −a  Khi đó x2 ≡ -a (mod p)    = 1.  p 45


Vậy có điều phải chứng minh.

L

Hay phương trình x2 + py + a = 0 có nghiệm nguyên (x, -t).

FI CI A

 −a  Ngược lại, nếu   = 1 thì tồn tại x sao cho x2 ≡ - a (mod p)  x2 + a = pt với t  p nguyên.

Ví dụ 14. Chứng minh rằng mọi ước nguyên tố của số nguyên có dạng x2 + x + 1, (x ∈ ℤ ) hoặc bằng 3 hoặc có dạng 3k + 1. Nếu x = 1 thì có p = 3 là ước của x2 + x + 1.

OF

Lời giải

Xét p > 3, ta có x2 + x + 1 ≡ 0 (mod p) ⇔ (2x + 1)2 ≡ -3 ( mod p).

ƠN

 −3  Hay   = 1 .  p p −1  −3   −1  3  3 Mà   =    = (−1) 2 ( ). p  p   p  p 

NH

p −1  −3   −1  3  3 3 3 Nếu p = 12k + 5 thì ( ) = - 1    =    = (−1) 2 ( ) = ( ) = −1 (loại). p p p  p   p  p  p −1  −3   −1  3  3 3 3 Nếu p = 12k – 1 thì ( ) = 1    =    = (−1) 2 ( ) = −( ) = −1 (loại). p p p  p   p  p 

QU Y

Vậy p = 12k + 1 hoặc p = 12k + 7.

Cả hai trường hợp trên đều có dạng 3k + 1. Từ đó có điều phải chứng minh. n

Ví dụ 15. Cho k = 22 + 1 với n nguyên dương. Chứng minh rằng k là số nguyên tố khi k−1

Lời giải

M

và chỉ khi k là ước của 3 2 + 1.

k−1

Y

Nếu k là ước của 3 2 + 1 thì ta có k−1

3 2 ≡ -1 (mod k)

(1)

⇔ 3k -1 ≡ 1 (mod k)

(2)

DẠ

Gọi d là bậc của 3 modulo k Từ (1) và (2) ta có d | k – 1 nhưng d lại không chia hết 46

k −1 2


 d = k – 1  k là số nguyên tố.

3 k ( )=( ) k 3 2 3

k 3

FI CI A

Ta có k là số nguyên tố dạng 4l + 1 nên theo luật tương hỗ Gauss ta có

L

Ngược lại, k là số nguyên tố

Mà k ≡ 2 (mod 3) nên ( ) = ( ) = −1 (do 2 không phải số chính phương mod 3). Từ đó suy ra k −1 2

≡ −1(mod k ) ⇔ 3

k −1 2

+ 1 ≡ 0(mod k ) .

đpcm.

OF

3

ƠN

Ví dụ 16. Chứng minh rằng với mọi n nguyên dương, số 2n + 1 không có ước nguyên tố dạng 8k + 7.

Lời giải

NH

Giả sử tồn tại số nguyên tố p = 8k + 7 sao cho p | 2n + 1 Nếu n chẵn, ta có

2n ≡ -1 (mod p)

Do đó

QU Y

Suy ra -1 là số chính phương mod p

1= (

Nếu n lẻ, ta có

p −1 −1 ) = (−1) 2 = (−1)4 k +3 = −1 (vô lí) p

M

2n ≡ -1 (mod p)  2n + 1 ≡ -2 (mod p)

Suy ra -2 là số chính phương mod p

Do đó, ta có

1= (

−2 −1 2 ) = ( )( ) = −1 .1 = -1. (vô lí) p p p

2.2.1 Các ví dụ nâng cao

DẠ

Y

Vậy ta có điều phải chứng minh.

Trong phần này chúng tôi cố gắng cập nhật các bài toán mới nhất, xuất hiện trong các đề thi gần đây và hướng giải quyết tinh tế và phức tạp hơn, từ đó giúp học sinh rèn kĩ năng ứng dụng thặng dư bậc hai một cách linh hoạt. 47


i

i =1

  p  = 0. 

Lời giải Gọi g là một căn nguyên thủy (mod p) Khi đó ta có {1, 2, ..., p - 1} = {g0, g1 , ..., gp - 2} (trong mod p)

 i  p −2  g i     =   i =1  p  i =0  p  p −1

 gi 

 gi 

Từ đó suy ra điều phải chứng minh.

ƠN

Dễ thấy   = 1 nếu i chẵn và   = - 1 nếu i lẻ  p  p

OF

Do đó

FI CI A

p −1

L

Ví dụ 17. Cho p là một số nguyên tố lẻ. Chứng minh rằng

NH

Ví dụ 18. (IMO Shortlist 2020) Với mỗi số nguyên tố lẻ p cho trước, kí hiệu

d p ( n) ∈ {0;1;2;...; p − 1} là số dư trong phép chia n cho p . Ta gọi ( a0 , a1 , a2 ,...) là một p − dãy, nếu ( ao , p ) = 1 và an+1 = an + d p ( an ) với mọi n ≥ 0 .

QU Y

a) Tồn tại hay không vô hạn các số nguyên tố p thỏa mãn tìm được hai p − dãy

( a0 , a1 , a2 ,...) và ( b0 , b1 , b2 ,...) sao cho an > bn với vô hạn n , bm > am

với vô hạn m ?

b) Tồn tại hay không vô hạn các số nguyên tố p thỏa mãn tìm được hai p − dãy

( a0 , a1 , a2 ,...) và ( b0 , b1 , b2 ,...) sao cho a0 > b0

M

Lời giải

nhưng an < bn với mọi n ≥ 1 ?

Để thuận tiện cho việc trình bày, ta thay kí hiệu d p ( n) bởi kí hiệu r ( n) .

Vì p là số nguyên tố lẻ nên đặt d = ord p 2 thì dễ thấy d | p − 1 .

(

)

Với mỗi p − dãy ( a0 , a1 , a2 ,...) ta thấy ngay r ( an ) = r 2n a0 .

(

)

Y

Vì r 2d a0 = r ( a0 ) nên {r ( an )} tuần hoàn với chu kì d .

DẠ

Lại có d | p − 1 nên với mọi n thì p−2

an+ p −1 − an =  r ( 2i a0 ) i =0

48


và d −1

L

a n + d − a n =  r ( 2 i a0 )

FI CI A

i =0

là một hằng số với mỗi p − dãy ( a0 , a1 , a2 ,...) .

a) Chọn p là một số nguyên tố thỏa mãn p ≡ 5[8] . Theo nguyên lý Dirichlet tồn tại vô

số số nguyên tố p có dạng này. Ta xét hai p − dãy {an } và {bn } với các số hạng đầu

tiên là a0 = 1 , b0 = −1 và chứng minh chúng thỏa mãn yêu cầu đề bài. Vì 2 không phải

OF

thặng dư bình phương mod p nên p −1 2 2  p  ≡ 2 ≡ −1 (mod p ).  

ƠN

Do đó

   p −1  r ( ai ) + r  a p −1  = r ( ai ) + r  2 2 ai  = r ( ai ) + r ( − ai ) = p    i+ 2 

NH

với mọi i. Từ đây ta có

p −3 2

p−2

 r (a ) =  r (a ) + r  a i

i =0

QU Y

i =0

i

p −1 i+ 2

 p ( p − 1) = 2 

là một hằng số không phụ thuộc vào a0 . Chứng minh tương tự với dãy {bn } và từ đó suy ra

an+ p −1 − an = bn + p −1 − bn =

p ( p − 1) . 2

M

Hay an + p −1 − bn + p −1 = an − bn với mọi n .

Từ đó ta thấy {an − bn } cũng là dãy tuần hoàn với chu kì p − 1 . Lại có a0 > b0 và

a1 = 2 < p − 2 = b1 nên ak ( p −1) > bk ( p −1) với mọi k ∈ ℕ và ak ( p −1)+1 > bk ( p −1)+1 với mọi

k ∈ℕ .

Y

b) Chọn p là một số nguyên tố thỏa mãn p ≡ 7(mod 8). .

DẠ

Theo nguyên lý Dirichlet tồn tại vô số số nguyên tố p có dạng này.

Ta xét hai p − dãy {un } và {vn } với các số hạng đầu tiên là u0 = pA + 1 , v0 = pB − 1 với A, B là các số nguyên nào đó. 49


Để ý rằng

FI CI A

p −1 2 2  p  ≡ 2 ≡ 1 (mod p ).  

L

Vì 2 là thặng dư bình phương mod p nên

p −1 là số lẻ nên không tồn tại n nguyên dương nào thỏa mãn 2 2n ≡ −1 (mod p). p −1 nên d là số lẻ. 2

OF

Hơn nữa dễ thấy d là ước của

Do đó các số hạng của dãy {r ( u n )} không trùng với bất cứ số hạng nào của dãy

{r ( v )} . n

ƠN

Vì vậy các số hạng của {un } cũng không trùng với bất cứ số hạng nào của {vn } . Với nhận xét từ đầu bài ta đặt

d −1

d −1

S1 = un+ d − un =  r ( 2 u0 ) =  r ( 2i ) ;

NH

i

i =0

i =0

d −1

d −1

i =0

i =0

S2 = vn+ d − vn =  r ( 2i v0 ) =  r ( −2i ).

QU Y

Ta có

d −1

d −1

S1 + S 2 =  r ( 2 ) +  r ( −2i ) = pd i

i =0

i =0

là một số lẻ (vì p , d là các số lẻ). Do đó trong hai số S1 , S 2 có một số lớn hơn số còn lại. Không mất tính tổng quát, giả sử S1 > S 2 .

M

Thay vào ta được

un+ d − un > vn + d − vn  un + d − vn+ d > un − vn .

Như vậy hiệu {un − vn } là một dãy tăng ngặt. Do đó, kể từ một giá trị n đủ lớn nào đó thì ta luôn có un > vn .

Y

Bây giờ chọn A, B là các số nguyên sao cho u0 < v0 .

DẠ

Tồn tại chỉ số M ∈ ℕ thỏa mãn uM < vM và un > vn với mọi n > M . Xét hai p − dãy

{an } và {bn } thỏa mãn a0 = vM

và b0 = uM thì đây chính là hai dãy thỏa mãn đề bài.

50


Ví dụ 19 (EMC 2020) Cho p là một số nguyên tố. An và Bình cùng chơi một trò chơi

L

như sau: An viết một số nguyên dương X lên bảng, sau đó đưa cho Bình một dãy số nguyên dương ( an )n∈ℕ . Tới lượt Bình, cậu tiến hành thực hiện một chuỗi các biến đổi

FI CI A

theo quy tắc sau: ở lần biến đổi thứ n , xóa số Y đang được viết trên bảng và thay nó bởi số Y + an hoặc Y .an . Bình thắng cuộc nếu như tại một bước nào đó, trên bảng xuất hiện một số là bội của p . Hỏi rằng bất kể ban đầu An chọn X và ( an )n∈ℕ như thế nào, liệu Bình có thể thắng được hay không nếu: a. p = 109 + 7

OF

b. p = 109 + 9

Lời giải

a) Câu trả lời là không. Ta sẽ chỉ ra cách An chọn X và ( an )n∈ℕ để Bình không thể

ƠN

thắng.

Kí hiệu yn là số xuất hiện ở trên bảng sau bước thứ n .

Chọn X là một số nguyên dương thỏa mãn X ≡/ 0, X ≡/ 1 (mod p ). −1

NH

Chọn a1 = X .( X − 1) −1 , trong đó ( X − 1)

là phần tử nghịch đảo của ( X − 1) trong

hệ thặng dư thu gọn mod p (lưu ý vì X − 1 ≡/ 0 [ p ] nên X − 1 nằm trong hệ thặng dư thu gọn mod p và do đó có phần tử nghịch đảo ( X − 1) −1 ).

QU Y

Tại bước thứ nhất: sau khi xóa số X Bình sẽ điền lên bảng số X + a1 hoặc X .a1 . Ta có

X X2 = (mod p); X −1 X −1 X X2 X .a1 ≡ X . = (mod p). X −1 X −1

M

X + a1 ≡ X +

Như vậy dù Bình chọn cách nào trong hai cách thì cũng đều thu được số y1 ≡

X2 X2 ≡/ 0 (mod p ), ≡/ 1 (mod p). X −1 X −1

Y

chứng minh Thật vậy

DẠ

X2 Vì X ≡/ 0, X ≡/ 1 (mod p ) nên hiển nhiên y1 ≡ ≡/ 0 (mod p). X −1

51

X2 Ta X −1


X2 ≡ 1 (mod p). X −1

L

Giả sử

FI CI A

Suy ra

 −3  2 X 2 − X + 1 ≡ 1 (mod p )  ( 2 X − 1) ≡ −3 (mod p )    = 1.  p  Điều này vô lý vì p là số nguyên tố chia 3 dư 2.

OF

X2 Do đó y1 ≡ ≡/ 1 (mod p ). X −1 −1

Vì y1 ≡/ 0, y1 ≡/ 1 (mod p ) nên có thể chọn a2 = y1 ( y1 − 1) .

Chứng minh tương tự như trên thì sau bước thứ hai trên bảng sẽ xuất hiện y2 mà

ƠN

y2 ≡/ 0, y2 ≡/ 1 (mod p).

Tiếp tục quá trình này ta luôn có thể kiểm soát được số dư khi chia cho p của các yn xuất hiện trên bảng sau mỗi bước là duy nhất và đảm bảo rằng yn ≡/ 0, yn ≡/ 1 (mod p ).

NH

Tổng quát hóa quá trình trên chúng ta có dãy ( an )n∈ℕ được xây dựng như sau:

QU Y

a0 = X  −1 a1 = X ( X − 1)  −1 an+1 = zn ( zn − 1) ∀n ≥ 1

Trong đó z n là số nguyên được chọn sao cho

an2 zn ≡ ≡ yn an − 1

M

với yn chính là số sẽ xuất hiện trên bảng sau bước thứ n . Sử dụng chứng minh ở trên

ta thấy ngay

yn ≡/ 0 (mod p).

Như vậy Bình sẽ không bao giờ thắng cuộc.

Y

b) Câu trả lời cũng là không.

DẠ

Ta cũng chỉ ra cách An chọn X và ( an )n∈ℕ để Bình không thể thắng. Chọn X là số nguyên dương thỏa mãn phương trình đồng dư

2 X 2 − 2 X + 1 ≡ 0 (mod p). 52


Phương trình này tương đương với (2 X − 1) 2 ≡ −1 (mod p ).

trình trên có nghiệm. Ngoài ra dễ thấy

X ≡/ 0, X ≡/ 1 (mod p ).

FI CI A

L

Vì p = 109 + 9 ≡ 1(mod 4) nên -1 là thặng dư bình phương mod p và do đó phương

Chọn a1 , a2 giống như phần a. Ta chứng minh y2 ≡ X (mod p ). Tương tự phần a ta có

OF

X2 y1 ≡ (mod p). X −1

Lập luận tương tự để thu được y1 ≡/ 0, y1 ≡/ 1 (mod p ). Ta có:

ƠN

X2 y X2 . a2 ≡ 1 ≡ X2 − 1 = 2 X y1 − 1 X − X +1 −1 X −1

NH

T ừ đó

X4 y2 ≡ ( X − 1)( X 2 − X + 1) .

QU Y

Lại có:

X≡

X4 (mod p) ( X − 1)( X 2 − X + 1)

⇔ X 3 ≡ X 3 − 2 X 2 + 2 X − 1 (mod p)

M

⇔ 2 X 2 − 2 X + 1 ≡ 0 (mod p ).

a1 = X ( X − 1)−1  a = X 2 ( X 2 − X + 1)−1  2 a2 k +1 = a1∀k ≥ 1  a2 k = a2∀k ≥ 2

DẠ

Y

Điều này luôn đúng do cách chọn X . Vậy ta chỉ cần chọn ( an )n∈ℕ thỏa mãn:

Thì các số Bình viết trên bảng khi chia cho p sẽ luôn có số dư trùng với số dư của y1 hoặc y2 và do đó, Bình không thể giành chiến thắng. 53


L

Ví dụ 20. Cho p là số nguyên tố lẻ lớn hơn 3, p ≡ 1 (mod 6). Chứng minh rằng tồn tại các số nguyên x, y sao cho x2 + 3y2 = p.

FI CI A

Lời giải  −3  Vì p ≡ 1 (mod 6) nên   = 1.  p

Khi đó tồn tại r nguyên dương sao cho r2 ≡ -3 (mod p).

Đặt f(u, v) = u + rv và k =  p  .

Xét tất cả các cặp (u, v) với 0 ≤ u ≤ k và 0 ≤ v ≤ k. Có tất cả (k + 1)2 cặp phân biệt.

OF

Khi đó k <  p  < k + 1 do p là số nguyên tố.

ƠN

Mà (k + 1)2 > p nên theo nguyên lý Dirchlet tồn tại (u1, v1) ≠ (u2, v2) sao cho f(u1, v1) ≡ f(u2, v2) (mod p)

⇔ u1 + rv1 ≡ u2 + rv2 (mod p) ⇔ (u1 – u2) ≡ - r(v1 – v2) (mod p).

NH

Hay tồn tại a, b nguyên không đồng thời bằng 0 sao cho

a ≡ -rb (mod p)  a2 ≡ r2b2 (mod p)  a2 ≡ -3b2 (mod p). ⇔ a2 + 3b2 ≡ 0 (mod p). Mà 0 ≤ |u1 - u2| ≤ k và 0 ≤ |v1- v2 | ≤ k nên a2 + 3b2 ≤ 4k2 < 4p.

QU Y

Từ đó suy ra a2 + 3b2 = p hoặc a2 + 3b2 = 2p hoặc a2 + 3b2 = 3p. Nếu a2 + 3b2 = 2p  a và b cùng tính chẵn lẻ Nếu a = 2m + 1 và b = 2n + 1 thì a2 + 3b2 ≡ 1 + 3 (mod 4) ≡ (0 mod 4). Nếu a = 2m, b = 2n thì a2 ≡ b2 ≡ 0 (mod 4)  a2 + 3b2 ≡ (0 mod 4).

M

Mà p lẻ suy ra 2p ≡ 2 (mod 4) vậy trường hợp này không thỏa mãn. Do đó a2 + 3b2 = p hoặc a2 + 3b2 = 3p.

Nếu a2 + 3b2 = 3p thì a phải chia hết cho 3 hay a = 3c. Suy ra (3c)2 + 3b2 = 3p ⇔ b2 + 3c2 = p. Từ đó suy ra điều phải chứng minh.

DẠ

Y

Ví dụ 21 (MMO 2021) Cho dãy số ( xn )n∈ℕ* được xây dựng như sau:

7   x1 = 2 .   xn +1 = xn ( xn − 2 )  54


a , trong đó a, b là các số nguyên dương nguyên tố cùng nhau. Chứng minh b rằng: nếu p là một ước nguyên tố của a thì p = 3 hoặc p ≡ 1 (mod 3) .

FI CI A

L

Đặt x2021 =

Lời giải 2 Ta sẽ chứng minh bằng quy nạp xn = 2

n −1

n −1

+ 2−2 + 1 với mọi n .

Thật vậy 0

0

x1 = 22 + 2−2 + 1.

OF

Giả sử mệnh đề đúng tới n = k ( k ≥ 1) . Tức là k −1

k −1

Ta có

(

k −1

)(

k −1

k −1

ƠN

xk = 22 + 2−2 + 1.

) (

k −1

k −1

xk +1 = xk ( xk − 2 ) = 22 + 2−2 + 1 22 + 2−2 − 1 = 22 + 2−2 Ta có

22020

+2

−22020

QU Y

x2021 = 2

NH

Vậy mệnh đề trên được chứng minh.

nên

a = 22

2 Đặt a = x 2 + x + 1, với x = 2

2020

+1 =

2021

+ 22

22

2020

2021

+ 22 22

2020

+1

2020

+ 1.

≡ 1 (mod3) .

Ta có

M

Từ đó dễ thấy a⋮3 . Gọi p > 3 là một ước nguyên tố của a .

a = x 2 + x + 1 ≡ 0 (mod p ) ⇔ 4 x 2 + 4 x + 4 ≡ 0 (mod p ) . 2

⇔ ( 2 x + 1) ≡ −3 (mod p ).

Y

Do đó −3 là số chính phương mod p .

DẠ

Điều này chỉ xảy ra khi p ≡ 1 (mod12) hoặc p ≡ 7 (mod12) . Cả hai trường hợp này ta đều có p − 1⋮3 .

Ví dụ 22. Cho p là số nguyên tố lẻ, kí hiệu 55

k −1

)

2

k

k

− 1 = 22 + 2−2 + 1.


L

p  n(n + a)  S (a, p) =    p  n =1 

b) Chứng minh rằng p

 S (a, p) = 0; a =1

c) Chứng minh rằng nếu (a, p) = 1 thì S(a, p) = S(1, p); d) Chứng minh rằng S(a, p) = -1 nếu (a, p) = 1.

OF

Lời giải

FI CI A

a) Tính S(0, p);

 n2   n =1  p  p

a) Ta có S (0, p ) =    n2 

 p2 

  = p – 1. 

NH

 n2 n =1  p p

Từ đó suy ra S (0, p ) =  

ƠN

Mà   = 1 với mọi n ∈ {1, 2, …, p - 1} và   = 0  p  p 

b) Ta có

p p  n( n + a )  p p  n( n + a )  S ( a , p ) =     =    p  n =1 a =1  p  a =1 a =1 n =1  p

QU Y

p p  nb  =   , n + a ≡ b(mod p) n =1 b =1  p  p p  n  b  =      n =1 b =1  p   p  2

M

Mà trong {1, 2, ..., p - 1} có đúng

2

 n  p  b   p  b    p −1  b    p =       =      =      , do   = 0 n =1  p  b =1  p   p  b =1  p    b =1  p   p

số chính phương mod p nên

p −1 p −1 số là số chính phương mod p và không là 2 2

p −1

b

b =1

  p  = 0. 

phần c) và d) xin nhường cho bạn đọc

DẠ

Y

Từ đó có điều phải chứng minh.

Ví dụ 23. Cho p là một số nguyên tố và p > 3. Chứng minh rằng tổng các thặng dư (mod p) bằng 0 (mod p). Lời giải 56


Chúng ta biết rằng với mọi p lẻ, luôn có

p −1 thặng dư (mod p) (chứng minh này dựa 2

FI CI A

Đặt S =

L

vào căn nguyên thủy). k 2.

1≤ k ≤ p −1 ∃a∈ℤ , a 2 ≡ k (mod p )

Mà k2 ≡ (p - k)2 (mod p) do đó nếu k là một thặng dư (mod p) mà k > thay k2 bởi (p - k)2.

k

2

=

k =1

p( p 2 − 1) (mod p). 24

OF

V ậy S ≡

p −1 2

p −1 thì ta có thể 2

Mà p > 3  p2 ≡ 1 (mod 8) và p2 ≡ 1 (mod 3)  p2 – 1 ≡ 0 (mod 24) p2 −1 ∈ ℤ  S ≡ 0 (mod p) 24

ƠN

V ậy

Vậy có điều phải chứng minh.

NH

Ví dụ 24 (Baltic Way 2019)

Cho p là một số nguyên tố lẻ. Chứng minh rằng: với mỗi số nguyên c cho trước, tồn tại một số nguyên a thỏa mãn p +1 2

+ (a + c)

p +1 2

QU Y

a Lời giải.

≡ c (mod p ).

Nếu c⋮ p thì ta chỉ cần chọn a = 0 .

Nếu c ⋮ p thì ta xét hai trường hợp sau:

c  = 1 , chọn a = 0 .  p

Khi đó

M

TH1: c là thặng dư bình phương mod p , tức 

a

p +1 2

+ (a + c)

p +1 2

=c

p +1 2

=c

p −1 2

c .c ≡   c ≡ c (mod p ).  p

Y

c  = −1 , xét hai khả năng: p  

DẠ

TH2: c không là thặng dư bình phương mod p , tức 

 −1   = 1 . Chọn a = − c thì ta có p  

+ Nếu p ≡ 1 (mod 4) thì 

57


+ ( −c + c )

p +1 2

= (−c)

p −1 2

 −c   −1  c  (−c) ≡   (−c ) ≡    (−c) ≡ c (mod p ).  p   p  p 

L

( −c )

p +1 2

FI CI A

.

 −1   −c   −1  c  = −1 và do đó   =    = 1.   p  p   p  p 

+ Nếu p ≡ 3 (mod 4) thì 

Xét hệ thặng dư thu gọn mod p gồm các số {−c, −2c, −3c,..., −( p − 1)c} : trong p − 1

p −1 p −1 thặng dư bình phương mod p và số không phải thặng dư 2 2 bình phương mod p .

OF

số này có đúng

Chọn i ≥ 2 là số đầu tiên thỏa mãn −ic không phải thặng dư bình phương mod p . Khi Chọn a = −ic thì ta được:

( −ic)

p +1 2

+ ( −ic + c )

p +1 2

≡ (−ic )

ƠN

đó −(i − 1)c là thặng dư bình phương mod p .

p −1 2

( −ic) + ( − ( i − 1) c )

p −1 2

(− ( i − 1) c)

NH

≡ ( −1) (−ic ) + ( − ( i − 1) c) ≡ c (mod p ). Từ đó ta có điều phải chứng minh.

Ví dụ 25. Tìm x, n nguyên dương sao cho

x3 + 2x + 1 = 2n.

QU Y

Lời giải

Do x nguyên dương nên x3 + 2x + 1 ≥ 4  n ≥ 2. Nếu n = 2, ta dễ dàng tìm được x = 1. Xét n ≥ 3

M

(1) ⇔ x(x2 + 2) = 2n – 1 là số lẻ nên x lẻ Do đó, x2 + 2 ⋮ 3  2n ≡ (1 mod 3) ⇔ (-1)n ≡ 1 (mod 3).

Từ đó suy ra n chẵn.

DẠ

Y

Mặt khác

x3 + 2x + 1 = 2n. ⇔ x3 + 2x + 3 = 2n + 2. ⇔ (x + 1)(x2 –x + 3) = 2n + 2.

Giả sử p là một ước nguyên tố của x2 – x + 3  p lẻ Khi đó 58

(1)


2n + 2 ≡ 0 (mod p) ⇔ 2n ≡ -2 (mod p).

L

−2 ) = 1. p

Ta có 2

p −1 p −1 −2 −1 2 1 = ( ) = ( )( ) = (−1) 2 (−1) 8 . p p p

Từ đó suy ra p = 8k + 1 hoặc p = 8k + 3.

Do đó ta suy ra x3 + 2x + 1 ≡ 0 (mod 8). Mà x lẻ  x = 8k ± 1, x = 8k ± 3.

OF

Mà n ≥ 3  2n = 8.2n – 3 ≡ 0 (mod 8).

FI CI A

Mà n chẵn suy ra (

ƠN

Nếu x = 8k ± 1  x3 + 2x + 1 ≡ ( ± 1)3 + 2( ± 1) + 1 ≡ 0 (mod 8) (vô lí). Nếu x = 8k + 3 thì x3 + 2x + 1 cũng không thể chia hết cho 8. Nếu x = 8k + 5 thì thỏa mãn.

NH

Khi đó

x2 – x + 3 ≡ 52 – 5 + 3 ≡ 7 (mod 8). Hay

QU Y

x2 – x + 3 = 8l + 7.

Mà số dạng 8l + 7 thì không thể có ước nguyên tố dạng 8k + 1 hoặc 8k + 3. Do đó (1) vô nghiệm khi n ≥ 3. Vậy x = 1, n = 2.

Lời giải

M

Ví dụ 26. Tìm k ∈ ℕ sao cho ∃m : 2 k − 1| m 2 + 9.

Ta có đpcm ⇔ m 2 ≡ −9(mod 2 k − 1) . k

Giả sử p| 2 − 1

Y

 −9   −1  *Nếu p ≠ 3 có   =   = 1 nên p = 4n + 1 .  p  p

DẠ

Khi p= 4n+1 thì 2k − 1 ≡ 1(mod 4) nên k=1

*Nếu p=3 Đặt k = 2t s , s lẻ.  2 s − 1 không chia hết cho 3. Mà 2k − 1| m 2 + 9 nên

2 s − 1| m 2 + 9  s = 1 theo trường hợp trên. 59


Ta sẽ chứng minh với mọi k = 2t thoả mãn (*) bằng cách áp dụng định lý Trung hoa

FI CI A

Thật vậy: Ta thấy ( au , av ) = 1 ∀1 ≤ u ≠ v ≤ t vì giả sử tồn tại p mà ( au , av ) = q. Giả sử 2u −1 = 2k .2v −1

22

u −1

v −1

v −1

− 1 = (2 2 ) 2 k − 1 chia hết (2 2 ) 2 − 1. v −1

Mà (2 2 ) 2 − 1 = (2 2

v −1

− 1)(2 2

v −1

+ 1)

Theo Định lý thặng dư Trung Hoa tồn tại m sao cho

ƠN

m ≡ 0(mod a1 ) 0

m ≡ 3.2 2 (mod a2 ) …

t −1

suy ra m 2 ≡ 9.2 2 (mod at ) . t

NH

t −2

m ≡ 3.2 2 (mod at ) .

OF

nên av − 2 chia hết cho au . 2 chia hết cho q suy ra q =1 (mâu thuẫn).

L

i −1

cho bộ số ( a1 , a2 ,.., at ) với ai = 2 2 + 1.

Kết luận k = 2t .

QU Y

Vậy m 2 + 9 ≡ 9(2 2 −1 + 1)(mod a1a2 ...at ) .

Ví dụ 27. Cho p là số nguyên tố và a, b, c là 3 số nguyên bất kì. Chứng minh rằng tồn tại các số nguyên x, y, z không đồng thời chia hết cho p sao cho ax 2 + by 2 + cz 2

Lời giải

M

chia hết cho p.

TH1: Nếu một trong ba số a, b, c chia hết cho p thì bài toán hiển nhiên đúng. Bởi vì, chẳng hạn c ⋮ p, ta chỉ việc chọn (x, y, z) = (p, p, 1) thỏa mãn.

Y

TH2: Cả ba số a, b, c cùng không chia hết cho p, chọn z = 1. Bài toán trở thành ax 2 + by 2 ≡ −c (mod p )

DẠ

có nghiệm.

p +1 p +1 thặng dư bình phương (mod p) nên ax 2 , −c − by 2 đều có số dư khi 2 2 chia cho p khi x và y chạy khắp ℤ .

Do có

60


L

Nếu các số dư của ax2 luôn khác số dư của –c – by2 khi chia cho p thì vô lý vì khi đó có p + 1 số dư phân biệt.

FI CI A

Do đó phải tồn tại x, y ∈ ℤ sao cho ax 2 ≡ −c − by 2 (mod p ) .

Đến đây ta có điều phải chứng minh.

Nhận xét: Thực chất, bài toán này chỉ là một hệ quả trực tiếp của định lý Cauchy – Davenport! Ví dụ 28 (Iran TST 2020)

p −1 2

p −1 2

x ≡ x i

≡ ... ≡  xi

i =1

p −1 2

[ p].

i =1

ƠN

i =1

2 i

p −1 2

OF

p −1   Cho p là một số nguyên tố lẻ. Tìm tất cả các bộ  x1 , x2 ,..., x p −1  ∈ ℤ p2 thỏa mãn:  2 

Lời giải (dựa theo lời giải của TheUltimate123)

Với p = 3 thì mỗi tổng trên chỉ có một số hạng nên ta có thể chọn x1 tùy ý.

NH

Xét trường hợp p > 3 :

Kí hiệu T ⊂ {1;2;3;...; p − 1} là tập hợp các thặng dư bình phương mod p thì

T =

p −1 . 2

QU Y

Trước hết ta đi chứng minh bổ đề sau:

 (t − 1) x + 1   = 1 thì x ∈ {0;1} mod p . p  

Bổ đề: nếu với mọi t ∈ T ta có 

 (t − 1) x + 1   1   =   = 1 với mọi t ∈ T (thỏa mãn). p    p

M

Thật vậy: nếu x ≡ 0 [ p ] thì 

Nếu x ≡/ 0 [ p ] , do (t − 1) x + 1 là thặng dư bình phương mod p , xét ánh xạ:

f :T → T . t ֏ f (t ) = (t − 1) x + 1

Y

Vì x ≡/ 0 [ p ] nên f là một đơn ánh. −1

DẠ

Ngược lại với mỗi t ' ∈ t tồn tại t = ( t '− 1) x + 1 , trong đó x −1 là phần tử nghịch đảo

của x trong hệ thặng dư thu gọn mod p , sao cho f (t ) = t ' . Do đó f là một toàn ánh. 61


g p −1 − 1 t ≡ g = 2 (mod p ).  g −1 t∈T i =1 2i

(

)

FI CI A

p −1 2

L

Vậy f là một song ánh từ T vào chính nó. Đặt g là một căn nguyên thủy mod p thì

Vì g là căn nguyên thủy mod p nên g p −1 − 1 ⋮ p , lại có p > 3 nên g 2 − 1 ⋮ p , từ đó

 t ≡ 0 (mod p). Lại do nhận xét

f là một song ánh nên:

t∈T

 t =  (t − 1) x + 1 = x t + (1 − x ) t∈T

t∈T

OF

t∈T

p −1 ≡ 0 (mod p ) 2

p −1 ≡ 0 (mod p ) 2  x ≡ 1 (mod p ).  (1 − x )

ƠN

Bổ đề hoàn toàn được chứng minh.

Quay trở lại bài toán.

p −1

2

p −1 2

NH

Giả sử  x1 , x2 ,..., x p −1  ∈ ℤ p2 là một bộ thỏa mãn đầu bài. Đặt: p −1 2

p −1 2

a =  xi ≡  x ≡ ... ≡  xi i =1

QU Y

i =1

Với t ∈ T , đặt P( xi ) = ( (t − 1) xi + 1) Ta có

2 i

p −1 2 .

M

i =1

p −1 2

p −1 2

p −1 2

= 1 +  C pj −1 (t − 1) j xij . j =1

p −1 p −1 2 2

2 p −1 2

  p −1 p −1 +  C pj −1 (t − 1) j xij = +  C pj −1 (t − 1) j  x1j + x2j + ... + x pj −1  2 2 i =1 j =1 j =1 2 2  2 

  P( xi ) =

(mod p ).

i =1

P ( xi ) = ( (t − 1) xi + 1)

p −1 2

p −1 2

p −1

p −1

p −1

DẠ

Y

2 2 p −1 2 j p −1   P( xi ) = +  C p −1 (t − 1) j  xij ≡ + a  C pj −1 (t − 1) j (mod p) . 2 2 i =1 j =1 i =1 j =0 2 2 p −1 2

 p2−1  p −1   P ( xi ) = + a. t − 1 (mod p). 2 i =1   62


≡ 1 (mod p) , từ đó suy ra p −1 2

 P( x ) ≡ i

i =0

p −1 (mod p ). 2

Lại có, với mỗi i thì P( xi ) = ( (t − 1) xi + 1)

p −1 2

L

p −1 2

FI CI A

Vì t ∈ T nên t

chỉ có thể có số dư khi chia p là 0,1, −1

(lần lượt tương ứng với các trường hợp (t − 1) xi + 1 chia hết cho p , là thặng dư bậc hai mod p và không là thặng dư bậc hai mod p ).

OF

Do đó

  P ( x1 ) ≡ P ( x2 ) ≡ ... ≡ P  x p −1  ≡ 1 (mod p ).  2 

ƠN

Từ đó suy ra (t − 1) xi + 1 là thặng dư bậc hai mod p với mọi i và với mọi t ∈ T . Sử dụng bổ đề trên ta có được xi ∈ {0;1} mod p . Thử lại vào đẳng thức ở đầu bài thấy thỏa

DẠ

Y

M

QU Y

NH

mãn.

63


L

Chương 3. Hệ số nhị thức và một số định lý cổ điển

FI CI A

3.1 Một số tính chất của hệ số nhị thức 3.1.1 Tính chất 1

Cho m, n nguyên dương, m < n và (m, n) = 1. Khi đó ta luôn có

Cnm ≡ 0(mod n). Chứng minh

Cnm =

OF

Ta có n m −1 Cn −1  mCnm = nCnm−−11 ≡ 0(mod n ). m

ƠN

Mà (m.n) = 1 nên Cnm ≡ 0 (mod n).

3.1.2 Tính chất 2

Cho p là một số nguyên tố. Khi đó với mọi i = 1, 2, …, p – 1 thì

3.1.3 Tính chất 3

NH

C ip ≡ 0(mod p ) .

Cho p là một số nguyên tố, k là một số nguyên dương, 1 ≤ k ≤ pk – 1. Khi đó ta có

QU Y

C ip k ≡ 0(mod p ) .

3.2. Một số định lý cổ điển khác 3.2.1 Định lý Sylvester

Cho các số nguyên dương a, b nguyên tố cùng nhau. Chứng minh rằng

M

N0 = ab – a – b

là số nguyên lớn nhất không biểu diễn được dưới dạng ax + by

với x, y là các số nguyên không âm. Hơn nữa, với mọi p, q nguyên, p + q = N0, có đúng một số biểu diễn được dưới dạng ax + by với x, y nguyên không âm.

Y

Chứng minh

DẠ

Bổ đề

Cho a, b nguyên dương nguyên tố cùng nhau, b > 1. Chứng minh rằng với mỗi số nguyên N, tồn tại duy nhất cặp số (x, y) sao cho N = ax + by, với 0 ≤ x < b.

Chứng minh bổ đề 64


a) Tính tồn tại

L

Theo định lý Bezout, luôn tồn tại x, y nguyên sao cho ax + by = N.

FI CI A

Chia x cho b ta được x = bq + x0, 0 ≤ x0 < b. Ta được

a(bq + x0) + by = n Û ax0 + b(aq + y) = N.

Û ax0 + by0 = N, y0 = aq + y.

(1)

Tính tồn tại được chứng minh.

OF

b) Tính duy nhất. Từ 0 ≤ x0 < b, ax0 = N – by0 º N (mod b).

Do (a, b) = 1 Þ {0, a, 2a, …, (b - 1)a} là hệ thặng dư đầy đủ mod b. Do đó tồ tại duy nhất x0 Î {0, 1, …, b - 1} sao cho ax0 º N (mod b).

ƠN

Suy ra số x0 xác định trong (1) là duy nhất. Do đó (x0, y0) duy nhất.

Trở lại định lý Sylvester

NH

a) Chứng minh với mọi n > N0, luôn biểu diễn được.

Do a, b nguyên tố cùng nhau nên tồn tại x, y sao cho 0 ≤ x < b sao cho n = ax + by Þ by = n – ax.

Þ by > N – ax = ab – a – b – ax ≥ ab – a – b – a(b - 1) = -b.

QU Y

Þ b(y + 1) > 0 Þ y > -1 Þ y ≥ 0.

Vây với mọi n > N0 thì luôn tồn tại x, y không âm sao cho n = ax + by. b) Giả sử tồn tại x, y không âm sao cho N = ax + by Mà theo trên ta có N = ab –a – b = a(b - 1) + (-1).b là sự biểu diễn duy nhất nên suy ra

M

x = b – 1; y = -1

Điều đó trái với giả thiết y ≥ 0.

Vậy có điều phải chứng minh.

3.2.2 Định lý Wolstenholme

Y

a) Cho p nguyên tố lẻ. Chứng minh rằng p −1

T ( p, 2) =  i =1

1 ⋮ p. i2

DẠ

b) Cho p ≥ 5, p nguyên tố. Đặt r 1 1 = 1 + + ... + , ( s, r ) = 1 . s 2 p −1 65


Chứng minh rằng r chia hết cho p2.

L

Chứng minh

FI CI A

a) Ta có

1 1 1 1 ( p − a)( p − b) + b( p − a) + a( p − b) + ab + + + ≡ (mod p) ab a( p − b) b( p − a) ( p − a)( p − b) ab( p − a )( p − b) p2 ≡ 0(mod p ) . ab( p − a )( p − b)

Do đó

1 1 ≡ 0(mod p )   ≡ 0(mod p), ( p, 4) = 1. 1≤i < j ≤ p −1 i. j 1≤i < j ≤ p −1 i. j

OF

4

i =1

p −1 p −1 1 12 1 12 ≡ ( ) − 2 ≡ ( ) (mod p )    2 i i =1 i 1≤i < j ≤ p −1 i. j i =1 i

ƠN

p −1

T ( p, 2) = 

Mặt khác p −1

1

1

p −1

1

1

p −1

1

p

i =1

NH

 i ≡ 2  ( i + p − i ) ≡ 2  ( i ( p − i ) ) ≡ 0(mod p) i =1

i =1

Vậy T(p, 2) º 0 (mod p).

QU Y

b) Ta có

p −1

p −1

p −1

2 1 2 1 1 1 A =  = ( + ) ≡ p ( )(mod p 2 ). − − i i p i i ( p i ) i =1 i =1 i =1

Điều phải chứng minh tương đương với p −1 2

1

M

 i( p − i) ≡ 0(mod p). i =1

Thật vậy, ta có

p −1 2

p −1

2 1 1 ≡ − 2 (mod p).  i =1 i ( p − i ) i =1 i

DẠ

Y

Do i2 º (p - i)2 (modp) nên p −1 2

p −1

p −1

p −1 2 1 1 1 2 1 1 1 ≡ − ≡ − ( + ) ≡ (mod p) ≡ 0(mod p).     2 2 2 2 2 i =1 i ( p − i) i =1 i ( p − i ) i =1 i i =1 i

Từ đó ta có điều phải chứng minh. 66


3.2.3 Định lý Legendre

L

Cho p là một số nguyên tố. Khi đó

FI CI A

+∞  n  n − s p (n) . v p (n!) =   k  = p −1 k =1  p 

(Phần chứng minh của định lý này bạn đọc có thể xem trong chuyên đề các bài toán về phân nguyên của cùng tác giả)

3.2.4 Định lý Lucas

OF

Cho m, n nguyên dương, m ≤ n, p là số nguyên tố. Biểu diễn của m, n trong hệ cơ số p là m = m0 + m1 p + ... + mk p k , n = n0 + n1 p + ... + nk p k

trong đó 0 ≤ ni, mi ≤ p – 1. mk và nk không đồng thời bằng 0. Khi đó

ƠN

Cnm = Cnn00 ...Cnmkk (mod p ) .

Chứng minh Theo định lý Legendre, ta có k

k

Do đó n

n0 + n1 p +...+ nk p k

k

ni

k

= ∏ (1 + x) p  ≡ ∏ (1 + x p ) n (mod p).   i =0 i =0 i

QU Y

(1 + x) = (1 + x)

NH

(1 + x) p ≡ 1 + x p (mod p ) .

i

i

Đồng nhất hệ số của xm ta được điều phải chứng minh. 3.2.5 Định lý Dirichlet

Cho p là một số nguyên tố. Chứng minh rằng tập số nguyên tố dạng

Bổ đề

M

Chứng minh

pn + 1 là vô hạn.

q | f p ( x)  (q, m(m − 1)) = 1.

Y

Cho p là một số nguyên tố. Đặt fp(x) = xp – 1 + xp – 2 + … + 1. Với mỗi số nguyên dương m chia hết cho p. Tồn tại hay không một số nguyên q sao cho

DẠ

Chứng minh bổ đề Gọi q là một ước nguyên tố của fp(m). Ta có f(m) º 1 (mod m) Þ q không là ước của m 67


Vì nếu m ⋮ q thì

L

f(m) – 1 º 0 (mod m) Þ f(m) – 1 º 0 (mod q) Þ -1 º 0 (mod q).

Do đó (q, m) = 1. Giả sử m º 1 (mod q). Khi đó

FI CI A

Dẫn đến điều vô lý.

f(m) = mp – 1 + mp – 2 + … + 1 º 1 + 1 + … + 1 º p (mod q).

Mà p | m Þ q | m (vô lý vì m º 1 (mod q)).

OF

Þ p º 0 (mod q) Þ p = q.

Từ đó suy ra với mọi q là ước nguyên tố của f(m) thì (q, m(m - 1)) = 1.

Trở lại chứng minh định lý

ƠN

Ta giả sử tập các số nguyên tố dạng pn + 1 là hữu hạn. Khi đó gọi tất cả các số nguyên tố đó là p1, p2, …, pk. Xét đa thức

NH

fp(x) = xp – 1 + xp -2 + … + 1. Đặt m = p1p2…pkp Þ m ⋮ p. Theo bổ đề trên, ta có q là ước nguyên tố của f(m). Khi đó (q, m(m - 1)) = 1.

QU Y

Theo định lý Fermat nhỏ ta có mq – 1 º 1 (mod q).

Ta có

(m - 1)fp(m) = mp – 1 º 0 (mod q) Þ mp º 1 (mod q)

Khi đó

M

Giả sử (p, q - 1) = 1, theo định lý Bezout, tồn tại các số nguyên r, s sao cho (q - 1)r + ps = 1. m = m(q - 1)rmps º 1 (mod q).

Vô lý vì (m – 1, q) = 1

Y

Do đó

(q – 1, p) = p Þ q – 1 ⋮ p Þ q = pn + 1.

DẠ

Mà (q, m) = (q, p1p2…pkp) = 1 nên q không thuộc {p1, p2, …, pk} Từ đó suy ra tập các số nguyên tố dạng pn + 1 là vô hạn.

68


L

Nhận xét: Thực tế, đây chỉ là một trường hợp đặc biệt của định lý Dirichlet. Định lý tổng quát được phát biểu như sau:

Tuy nhiên, hiện tại định lý trên chưa có chứng minh sơ cấp!

3.3 Ví dụ áp dụng 3.3.1 Các ví dụ cơ bản

FI CI A

“Cho a, b là hai số nguyên dương nguyên tố cùng nhau. Khi đó tập số nguyên tố có dạng an + b là vô hạn”

OF

Ví dụ 1. Chứng minh rằng: ∀ m, n ∈ N ta có (2m)!(2n)! ⋮ m!n!(m+n)! Lời giải Gọi p là một số nguyên tố bất kì

ƠN

Số mũ của p trong phân tích chính tắc của m!n!(m+n)! là  m   n  m + n α =  ( k  +  k  +  k ) . k =1  p  p   p  ∞

NH

Số mũ của p trong phân tích chính tắc của (2m)!(2n)! là: ∞

 2m   2 n  +  k ) . k   p  p 

β =  ( k =1

 2m 

 2n 

m

n 

m + n

QU Y

Ta dễ dàng chứng minh được  k  +  k  ≥  k  +  k  +  k   β ≥ α  p  p  p  p   p  Từ đó có điều phải chứng minh.

Ví dụ 2. Chứng minh rằng ∀ n ∈ N thì 2n không chia hết được n!. Lời giải

M

Ta có n! = 2 m .q Trong đó (2, q) = 1.

Ta phải chứng minh n > m. Theo định lý Legendre ta có: n  n 

n

m =   +  2  + ... +  p  với 2p ≤ n < 2p+1. 2 2  2 

DẠ

Y

Mặt khác

n  n  n  2  +  22  + ... +  2 p  ≤

n  n n n  2 + 2 2 + ... + 2 p  =  n − 2 p  < n .

Từ đó suy ra điều phải chứng minh. 69


(1 + x) p −2 = 1 + a1 x + a2 x 2 + ... + a p −2 x p − 2 .

Lời giải Ta có

ak + (−1)k +1 (k + 1) = C pk − 2 + (−1) k +1 (k + 1)

FI CI A

Chứng minh rằng ak + ( −1) k +1 ( k + 1) chia hết cho p với mọi k = 1, 2, …, p – 2.

( p − 2)( p − 3)...( p − 1 − k ) + (−1) k +1 (k + 1) k! (−2)(−3)...(−k − 1) ≡ + (−1) k +1 (k + 1) k! k (−1) (k + 1)! ≡ + (−1) k +1 (k + 1) ≡ 0(mod p ). k! Từ đó có đpcm.

a)

1008

 kC

k 2017

≡ 0(mod 2017 2 );

k =1

b)

504

 (−1) C k

k 2017

NH

Ví dụ 4 (VMO 2017). Chứng minh rằng

ƠN

OF

=

≡ 3(2 2016 − 1)(mod 2017 2 ).

Lời giải Dễ thấy

QU Y

k =1

k k −1 kC2017 = 2017C 2016 .

Do đó 1008

 kC

k 2017

M

k =1

1007

k = 2017  C2016 = k =0

2017 2016 k 2017 2016 1008 1008 (  C2016 − C2016 )= (2 − C2016 ). 2 k =0 2

1008 Điều phải chứng minh tương đương với 22016 − C2016 º 0 (mod 2017).

Theo định lý Fermat nhỏ, ta có 22016 – 1 º 0 (mod 2017). Mặt khác

1009.1010...2016 − 1008! (2017 − 1)(2017 − 2)...(2017 − 1008) − 1008! ≡ 1008! 1008! ≡ 0(mod 2017).

DẠ

Y

1008 C2016 −1 ≡

Từ đó ta có điều phải chứng minh.

b) Xét p nguyên tố bất kì ta có 70

L

Ví dụ 3 (Bình Dương 2020) Cho p là một số nguyên tố lẻ. Giả sử


p! p ( p − 1)( p − 2)...( p − (k − 1)) p ≡ . ≡ (−1) k −1 (mod p 2 ). ( p − k )!k ! k 1.2...(k − 1) k

L

C pk ≡

504

 (−1)

k

504

k C2017 ≡  ( −1) k ( −1) k −1

k =1

k =1

504 2017 1 (mod 2017 2 ) ≡ −2017  (mod 2017 2 ). k k =1 k

Điều phải chứng minh tương đương với 1 3(22016 − 1) (mod 2017) ≡ 2017 k =1 k

504

− 1 k =1 k

1 1 1 3

2n 1 1 1 1 (−1)k −1 − ( + + ... + ) =  = S2n − Sn . 2n − 1 2 4 2n k k =1

OF

n

Đặt Sn =   + + ... +

FI CI A

Từ đó ta có

Ta có

2016 (−1) k −1 (−1)k −1 1008 (−1) k −1 = S 2016 −  − k k k k =1 k =1 k =1

1008

ƠN

S504 = S1008 − 

Mặt khác

NH

1008 1008 1 1 2017 S 2016 ≡  ( − ) ≡ ( ) ≡0(mod 2017) . 2017 − k k =1 k (2017 − k ) k =1 k

Do đó điều phải chứng minh tương đương với

(−1) k −1 2016 (−1)k −1 1008 (−1) k −1 ≡ − ≡ 0(mod 2017) k k k k =1 k =1 k =1

1008

QU Y

S 2016 − 

k

k p

Do C ≡ (−1)

k −1

C p (−1)k −1 p 2 ≡ (mod p )  (mod p) nên k p k 1008 (−1)k −1 1008 (−1)k −1 1 2016 k k − ≡ (  C2017 −  C2017 ) k k 2017 k =1 k =1 k =1 k =1

2016

M

2017 3 3(22016 − 1) k (  C2017 −2) = (mod 2017). 2.2017 k =0 2017

Từ đó ta có điều phải chứng minh.

Y

Ví dụ 5. Chứng minh rằng 1 1 1 1 − + − ... + ≡ 0(mod1979). 1 2 3 1319

DẠ

Lời giải n

1 k =1 k

1 1 1 3

Đặt S n =   + + ... +

2n 1 1 1 1 ( −1) k −1 − ( + + ... + ) =  = S2n − Sn . 2n − 1 2 4 2n k k =1

Do đó 71


Từ đó ta có điều phải chứng minh.

2p .Chứng minh rằng  3 

Ví dụ 6. Cho p là một số nguyên tố, p ≥ 5. Đặt k =  k

C

i p

≡0(mod p 2 ) .

i =1

OF

Lời giải

Ta chứng minh cho trường hợp p = 3n + 2, khi đó k = 2n + 1. Ta có i p

k

≡ p

i =1

i =1

Điều phải chứng minh tương đương với

( −1) k −1 (mod p2). k

ƠN

k

C

(−1) k −1 ≡ 0(mod p ).  k i =1

Ta có

NH

k

(−1)k −1 1 1 1 1 ≡ − + − ... +  k 1 2 3 2n + 1 i =1

QU Y

k

1 1 1 1 1 1 1 1 1 1 1 1 ≡ ( + + ... + ) − 2( + + ... + ) ≡ ( + + ... + ) − ( + + ... + ) 1 2 2n + 1 2 4 2n 1 2 2n + 1 1 2 n ≡

1 1 1 1 2 n +1 1 1 + + ... + ≡ ( + ) ≡ 0(mod p ) . n +1 n + 2 2n + 1 2 k = n +1 k 3n + 2 − k

M

Trường hợp còn lại tương tự. p −1

Ví dụ 7. Cho p nguyên tố lẻ. Đặt T ( p, k ) =  i =1

1 . Với k lẻ, chứng minh rằng ik

p −1

T ( p, k ) =  i =1

1 ⋮ p. ik

Y

Lời giải

DẠ

Ta có p −1

T ( p, k ) =  i =1

1 1 p −1 1 1 1 p −1 i k + ( p − i ) k 1 p −1 i k + (−i ) k ≡ ( + ) ≡ ( ) ≡ ) ≡ 0(mod p ).   ( i k 2 i =1 i k ( p − i ) k 2 i =1 i k ( p − i ) k 2 i =1 i k ( p − i ) k

Từ đó ta có điều phải chứng minh. 72

L

FI CI A

1 1 1 1 1 − + − ... + ≡ S1318 − S659 + 1 2 3 1319 1319 1319 1319 1 1 1 1 1 1319 1979 )≡  ( ) ≡ 0(mod1979) ≡  ≡ ( + 2 k = 660 k 1979 − k 2 k =660 k (1979 − k ) k = 660 k


Ví dụ 8. Cho p nguyên tố lẻ. Đặt

L

i =1

1 . ik

Biện luận số dư của T(p,k) khi chia cho k

Lời giải Với mọi i Î {1, 2, …, p - 1} ta có (i, p) = 1.

FI CI A

p −1

T ( p, k ) = 

Do đó luôn tồn tại i’ Î {1, 2, …, p – 1}, i’ ≠ i sao cho ii’ º 1 (mod p). Mặt khác nếu i’ º j’ (mod p) thì

OF

1 1 ≡ (mod p)  i ≡ j (mod p) . i j

Do đó số i’ như trên là xác định duy nhất.

p −1

T ( p, k ) = 

1 p −1 k 0(mod p), k ⋮ p − 1 ≡ i ≡  i k i =1 −1(mod p), k ⋮ p − 1.

NH

i =1

ƠN

Khi đó

Ví dụ 9 (Định lý Wolstenholme) Cho p ≥ 5, p nguyên tố. Đặt

QU Y

r 1 1 = 1 + + ... + , ( s, r ) = 1 . s 2 p −1

Chứng minh rằng r chia hết cho p2.

Lời giải Ta có

M

p −1

p −1

p −1

2 1 2 1 1 1 A =  = ( + ) ≡ p ( )(mod p 2 ). i i p − i i ( p − i ) i =1 i =1 i =1

Y

Điều phải chứng minh tương đương với p −1 2

1

 i( p − i) ≡ 0(mod p). i =1

DẠ

Thật vậy, ta có p −1 2

p −1

2 1 1 ≡ − (mod p).   2 i =1 i ( p − i ) i =1 i

Do i2 º (p - i)2 (mod p) nên 73


p −1

p −1

Từ đó ta có điều phải chứng minh.

FI CI A

p −1 2 1 1 1 2 1 1 1 ≡ − 2 ≡ −  ( 2 + ) ≡ (mod p) ≡ 0(mod p).   2 2 2 i =1 i ( p − i) i =1 i ( p − i ) i =1 i i =1 i

L

p −1 2

Ví dụ 10. Chứng minh rằng số nguyên dương chẵn n là số hoàn hảo khi và chỉ khi n = 2m – 1(2m - 1) trong đó m là số nguyên dương, m ≥ 2 và 2m – 1 là số nguyên tố.

OF

Lời giải Gọi T(n) là tổng ước dương của n

a) Xét 2m – 1 – 1 là số nguyên tố, chứng minh n là số hoàn hảo.

ƠN

Ta có T (n) = T (2m −1 (2m − 1)) = T (2m −1 )T (2m − 1) =

Vậy n là số hoàn hảo.

NH

b) Xét n là số hoàn hảo.

2m − 1 .(1 + 2m − 1) = 2n 2 −1

Viết n = 2s.t, (t, 2) = 1. Do đó ta có

QU Y

T ( n ) = T (2 s )T (t ) = (2 s +1 − 1)T (t )

Mà n là số hoàn hảo nên

(2 s +1 − 1)T (t ) = 2n = 2 s +1 t .

Vì (2s+1 – 1, 2s +1 ) = 1 Þ T(t) ⋮ 2s+ 1 Þ T(t) = 2s + 1q.

M

Từ đó ta suy ra

(2s +1 - 1)T(t) = 2s + 1t Þ (2s+ 1 - 1)q = t.

Suy ra t ⋮ q.

Nếu t = q Þ 2s + 1 – 1 = 1 Þ s = 0 Þ n lẻ. Vậy t ≠ q.

Y

Mà t + q = 2s + 1q = T(t).

DẠ

Nếu q ≠ 1, mà t ⋮ q nên t có ít nhất 3 ước là t, q, 1 Þ T(t) ≥ t + q + 1 > t + q (vô lý).

Vậy q = 1 Þ T(t) = t + q = t + 1.

Nếu t không phải số thì t phải có ước số khác t và 1 Þ T(t) > t + 1 (vô lý). Vậy t = 2s + 1 – 1 và t là số nguyên tố. 74


Vậy n = 2s(2s + 1 - 1), 2s + 1 – 1 là số nguyên tố.

3.3.1 Các ví dụ nâng cao

FI CI A

L

Từ đó có điều phải chứng minh.

Ví dụ 11. Cho a là một số nguyên dương. Một số nguyên dương b được gọi là a-good b nếu Can − 1⋮ (an + 1) với mọi số nguyên dương n sao cho an ≥ b. Chứng minh rằng nếu b là a-good thì b là số chẵn.

Lời giải

OF

Gọi p là số nguyên tố lớn hơn cả a và b. Khi đó (a, p) = 1.

Do đó, tồn tại vô số số nguyên dương m sao cho am + 1 ≡ 0 (mod p). Chọn m sao cho am

b.

ƠN

Khi đó

b p | am + 1| Cam −1.

Ta có

( am)( am − 1)...( am − b + 1) (−1)( −2)...( −b) ≡ ≡ ( −1)b (mod p ) . b! b!

NH

b 1 ≡ Cam =

Do đó b là số chẵn.

QU Y

Ví dụ 12. Cho a là một số nguyên dương. Một số nguyên dương b được gọi là a-good b nếu Can − 1⋮ (an + 1) với mọi số nguyên dương n sao cho an ≥ b. Gọi b là một số nguyên dương sao cho b là a-good và b + 1 không là số nguyên tố. Chứng minh rằng mọi ước nguyên tố của b + 1 đều là ước của a.

Lời giải

M

Phản chứng.

Giả sử tồn tại p nguyên tố, p là ước của b + 1 nhưng không là ước của a.

Do b + 1 không là số nguyên tố nên b + 1 ≠ p, b + 1 – p chia hết cho p.

Đặt α = v p (b + 1 − p )  b + 1 − p ≡ rpα (mod pα +1 )  b + 1 ≡ p + rpα ( pα +1 ) . Với 1

r

p-1.

DẠ

Y

Ta có (a, p) = 1 nên tồn tạp vô số số nguyên dương m sao cho am +1 ≡ p (mod p α +1 ) ⇔ am ≡ p - 1 (mod p α +1 ).

Chọn m sao cho am

b.

Khi đó 75


FI CI A

L

  am  am + 1 − p   α +1  = pα +1  p   α   b  = b + 1 − p − rp   pα +1  pα +1  Mặt khác ta có

 am   b   am − b  Do đó  α +1  =  α +1  +  α +1  . +1 p  p   p 

Ta có với mọi k

≥ 0, ta có

 am   b   am − b   pk  ≥  pk  +  pk       

OF

 am − b   (am + 1 − p ) − (b + 1 − p − rpα )  (am + 1 − p ) − (b + 1 − p − rpα ) − 1. =  pα +1  =  pα +1 pα +1    

ƠN

 v p (( am)!) ≥ v p (b !) + v p (( am − b)!) + 1 p  Cam ≡ 0(mod p ).

NH

p Mà p | am + 1| Cam − 1  p | 1 (vô lý).

Vậy ta có đpcm.

Nhận xét: Từ hai ví dụ trên, ta dễ dàng chứng minh được bài IMO SL 2019 sau

QU Y

Ví dụ 13 (IMO SL 2019) Cho số nguyên dương a. Chúng ta nói một số nguyên dương b b là a-good nếu Can − 1⋮ (an + 1) với mọi số nguyên dương n sao cho an ≥ b. Giả sử

rằng b là một số nguyên dương, b là a-good và b + 2 không là a-good. Chứng minh rằng b + 1 là một số nguyên tố. Lời giải

Dễ dàng suy ra từ ví dụ 11 và ví dụ 12.

M

Ví dụ 14 (Đề luyện ĐT TST 2020) Cho p là một số nguyên tố. Với mỗi số nguyên

dương k , đặt

Ak = 1k + 2k + ... + ( p − 1) k .

Chứng minh rằng:

Y

a) Nếu p ≥ 7 và k là một số nguyên dương lẻ sao cho 3 < k < p − 1 thì kpAk −1 − 2 Ak

DẠ

chia hết cho p 4 .

5 2 b) Nếu p ≥ 5 thì p Ap −1 − 2 Ap chia hết cho p .

Lời giải. 76


a) Xét k là một số nguyên dương lẻ tùy ý thì:

k

L

Ak = 1k + 2k + ... + ( p − 1) k k

k

k

i =0

i =0

FI CI A

= [ p − ( p − 1) ] + [ p − ( p − 2) ] + ... + ( p − 1) k

k

=  Cki ( −1)i ( p − 1)i p k −i +  Cki (−1)i ( p − 2)i p k −i + ... +  Cki (−1)i p k −i i =0

k

p −1

i =0

j =1

=  Cki ( −1)i p k −i  j i

≡ Ck3 p 3 Ak −3 − Ck2 p 2 Ak −2 + Ck1 pAk −1 − Ak (mod p 4 ) .

OF

 kpAk −1 − 2 Ak ≡ Ck2 p 2 Ak −2 − Ck3 p 3 Ak −3 (mod p 4 ). (*) Ta chứng minh hai bổ đề sau:

Bổ đề 1: nếu k là số nguyên dương nhỏ hơn p − 1 thì p | Ak .

ƠN

Chứng minh

{

}

Giả sử a là một căn nguyên thủy của p . Khi đó a; a 2 ;...; a p −1 là một hệ thặng dư thu

k

NH

gọn mod p và do đó: k

k

k

2k

Ak = 1 + 2 + ... + ( p − 1) ≡ a + a + ... + a

( p −1) k

a pk − a k = k . a −1

mà a

pk

QU Y

k Vì k là số nguyên dương nhỏ hơn p − 1 nên a ≡/ 1[ p ] .

(

− a k = a k a(

p −1)k

)

− 1 ⋮ p nên p | Ak .

Bổ đề 2: nếu k là số nguyên dương lẻ thỏa mãn 3 < k < p − 1 thì p 2 | Ak . Chứng minh

M

Vì k lẻ nên theo (*) thì

2 Ak ≡ kpAk −1 − Ck2 p 2 Ak −2 + Ck3 p 3 Ak −3 (mod p 2 ) .

Kết hợp với mệnh đề 1 ta có p 2 | Ak . Lại áp dụng mệnh đề 2 cho (*) ta thu được kpAk −1 − 2 Ak chia hết cho p 4 .

DẠ

Y

b) Với p là một số nguyên tố lẻ ta có

77


Ap = 1p + 2 p + ... + ( p − 1) p p

p

p

p

i =0

i =0

L

= [ p − ( p − 1) ] + [ p − ( p − 2) ] + ... + ( p − 1) p p

FI CI A

=  C ip (−1)i ( p − 1)i p p −i +  C ip ( −1)i ( p − 2)i p p −i + ... +  C pi (−1)i p p −i i =0

p

p −1

i =0

j =1

=  C ip ( −1)i p p −i  j i

≡ −C p4 p 4 Ap −4 + C 3p p 3 Ap −3 − C p2 p 2 Ap − 2 + C 1p pAp −1 − Ap (mod p 5 )

OF

 p 2 Ap −1 − Ap ≡ C p4 p 4 Ap − 4 − C 3p p 3 Ap −3 + C p2 p 2 Ap − 2 (mod p 5 ) .

2 3 4 Vì C p , C p , C p đều chia hết cho p mà theo mệnh đề 1 thì Ap −3 ⋮ p và theo mệnh đề 2

2

4

4

3

3

2

2

5

2

thì Ap −2 ⋮ p nên C p p Ap −4 − C p p Ap−3 + C p p Ap −2 ⋮ p ; từ đó dẫn tới p Ap −1 − 2 Ap

ƠN

chia hết cho p 5 .

Ví dụ 15 (IMO Shortlist 2013). Cho p là một số nguyên tố lẻ. Với mỗi số nguyên

dương

a , ta đặt

m * với m, n∈ℕ . Chứng minh rằng n

QU Y

Đặt S3 + S4 − 3S 2 =

NH

a a2 a p−1 Sa = + + ... + . 1 2 p −1

Lời giải

p| m.

Trước hết ta chứng minh bổ đề

C pk p

k −1

k

(mod p).

k ∈ℕ,1≤ k ≤ p −1.

Thật vậy

M

với p là một số nguyên tố lẻ,

( −1) ≡

C pk

p! ( p − k + 1)...( p − 1) (−k + 1)(−k + 2)...(−1) ( −1) = = ≡ = p k !( p − k )! p k! k! k

Y

Bổ đề được chứng minh.

DẠ

Trở lại bài toán

Sử dụng bổ đề trên ta được

78

k −1

(mod p).


FI CI A

Do đó ta có

L

k ak k −1 k a ≡ ( −1) .Cp . (mod p) ,∀k ∈ℕ,1≤ k ≤ p−1. k p

p

p −1

S a ≡  ( −1)

k −1

k =1

p a − 1) − a p + 1 ak 1  ( p−k p k k .C . =  +1 − a +  ( −1) .C p .a  = (mod p ). p p p k =0  k p

. Thay vào ta được:

2

OF

2 p − 3p + 1 + 3p − 4 p + 1 − 3(1p − 2 p + 1) S3 + S4 − 3S2 ≡ p

ƠN

2 p − 2) ( 4.2 p − 4 p − 4 ≡ ≡− (mod p ). p p

Vì p là số nguyên tố lẻ nên theo định lý Fermat nhỏ thì 2 p ≡ 2 (mod p). p

− 2)

2

≡ 0 (mod p ).

p

V ậy S 3 + S 4 − 3 S 2 =

p| m.

m ≡ 0 (mod p ). n

QU Y

Hay

NH

(2 Do đó

Ví dụ 16 (CMO 2015)

Cho số nguyên dương k ≥ 2, chứng minh rằng có vô hạn số nguyên dương n thỏa mãn

M

Lời giải

n + k /| C2nn .

Xét các trường hợp sau: Trường hợp 1: k có ước nguyên tố lẻ p. Đặt t = vp (k ) > 0 và chọn n = p

DẠ

Y

Khi đó υ p ( n + k ) = t và

(

m

υ p C2pp

m

) = υ (( 2 p )!) − 2υ (( p )!) m

p

m

p

= 2 p m −1 + 2 p m − 2 + ... + 2 − 2 ( p m −1 + p m − 2 + ... + 1) = 0 n

m

Do đó n + k /| C2n với n = p . 79

m

với

m> t.


Trường hợp 2: k = 2 m

m

m

−2 −4

m +1

) = υ (( 2

m +1

2

)

(

)

− 4 )! − 2υ 2 ( 2 m − 2 )!

FI CI A

(

υ 2 C22

L

Chọn n= 2 − 2 với m ∈ ℕ , m ≥ 2 thì n + k = 2 . Lại có:

= ( 2 m − 2 ) + ( 2 m−1 − 1) + ( 2 m− 2 − 1) + ... + 1 − 2 ( 2 m −1 − 1) + ( 2 m −2 − 1) + ... + 1 = ( 2 m − 2 ) − ( 2 m −1 − 1) + ( 2 m −2 − 1) + ... + 1 = m − 1 < m = υ2 ( n + k ) m

OF

= ( 2 m − 2 ) − ( 2 m − 2 − m + 1)

.

Vì vậy trong trường hợp này các số có dạng n= 2 − 2 thỏa mãn bài toán. t

Trường hợp 3: k = 2 với t ≥ 2 .

n= 2m với m>t . Khi đó υ2 ( n + k ) = t

(

υ 2 C22

m

m +1

ƠN

Chọn

) = υ ( ( 2 )!) − 2υ (( 2 )!) m +1

m

p

p

NH

= 2 m + 2 m −1 + ... + 1 − 2 ( 2 m −1 + 2 m − 2 + ... + 1) = 1 < t = υ2 ( n + k )

.

QU Y

m Vì vậy trong trường hợp này các số có dạng n= 2 thỏa mãn bài toán.

Ví dụ 17 (VMO 2016)

a) Chứng minh rằng n là số hoàn hảo lẻ thì n có dạng n= ps.m2

M

trong đó p là số nguyên tố dạng 4k + 1, s là số nguyên dương có dạng 4h + 1, m nguyên dương không chia hết cho p.

b) Tìm tất cả các số nguyên dương n > 1 sao cho n – 1 và Lời giải

Y

Kí hiệu hàm tổng ước dương của n là T(n).

DẠ

a) Xét phân tích chính tắc của n t

n = ∏ p1ai . i =1

Khi đó 80

n(n + 1) đều là số hoàn hảo. 2


piai +1 − 1 = 2n pi − 1

t

T ( n) = ∏

L

i =1

t

FI CI A

Ta lại có v2(2n) = 1; v2 (T ( n)) =  (v2 ( pia +1 − 1) − v2 ( pi − 1)) . i

i =1

Nếu a lẻ thì

v2(pa+1 - 1) – v2(p - 1) = v2(p - 1) + v2(p + 1) + v2(a + 1) – 1 – v2(p - 1) ≥ 1. Giả sử số đó là a1. Khi đó t

n = p1a1 ∏ piai = p1a1 m 2 .

OF

Do đó, trong các số ai, i = 1, 2, …, t chỉ có duy nhất một số lẻ.

ƠN

i=2

Ta cần chứng minh p1 = 4k + 1 và a1 = 4h + 1. Thật vậy

p1a1 +1 − 1 ) = v2 ( p1 + 1) + v2 (a1 + 1) − 1 ≥ 1, ∀p1 , a1 = 2l + 1. p1 − 1

NH

v2 (

Do đó

v2 ( p1 + 1) = 1  p1 = 4k + 1 p1a1 +1 − 1 ) =1⇔  ⇔ p1 − 1 v2 (a1 + 1) = 1  a1 = 4h + 1

QU Y

v2 (

Từ đó ta có điều phải chứng minh.

b) Trước tiên, ta chứng minh n là số lẻ. Thật vậy, giả sử n chẵn ta có n – 1 là số hoàn hảo lẻ. psm2 = (4 k + 1)s.(2l + 1)2 º 1 (mod 4).

Do đó

M

Theo a), mọi số hoàn hảo đều có dạng

n – 1 º 1 (mod 4) Þ n = 4k + 2 Þ

n( n + 1) = (2k + 1)(4k + 3) lẻ 2

DẠ

Y

Suy ra n(n + 1) = (2k + 1)(4k + 3) = psm2 2

Trong đó p º s º 1 (mod 4). Mà (2k + 1, 4k + 3) = 1 nên 4k + 3 chỉ có thể bằng p1s hoặc m2 hoặc ps.m2 81


Nhưng p1s º m2 º psm2 º 1 (mod 4) còn 4k + 3 º 3 (mod 4) (vô lý).

L

Vậy n lẻ Þ n – 1 chẵn Do đó ta có n – 1 = 2s(2s + 1 - 1). Suy ra n(n + 1) =  2 s (2 s +1 − 1) + 1  2 s −1 (2 s +1 − 1) + 1 2

+) Nếu s = 1 thì

FI CI A

Theo bài toán 1) mọi số hoàn hảo đều có dạng 2k(2k + 1 - 1).

OF

n( n + 1) =  2s (2 s +1 − 1) + 1  2 s −1 (2 s +1 − 1) + 1 = 28 2

Thoả mãn nên ta được n = 7. +) Nếu s > 1 thì

n( n + 1) =  2 s (2 s +1 − 1) + 1  2 s −1 (2s +1 − 1) + 1 là số hoàn hảo lẻ 2

ƠN

Do đó

 2s (2s +1 − 1) + 1  2s −1 (2s +1 − 1) + 1 = p a m2

NH

Do (2 s (2 s +1 − 1) + 1, 2 s −1 (2 s +1 − 1) + 1) = 1 nên một trong hai số này phải là số chính phương Ta sẽ chứng minh cả hai số này đều không là số chính phương. TH1: 2s(2s + 1 - 1) + 1 = b2 Þ 2s(2s + 1 - 1) = (b - 1)(b + 1)

QU Y

Do (b – 1, b + 1) | 2 và (b - 1)(b + 1) ⋮ 4 nên b – 1 và b + 1 đều chẵn. Mà 2s + 1 – 1 là số nguyên tố nên 2(2s + 1 - 1) phải là ước của b – 1 hoặc b + 1 Do s ≥ 2 Þ b > 2 Þ 2(b - 1) > b + 1 Suy ra

2(b - 1)(b + 1) > (b + 1)2 ≥ [2(2s + 1 - 1)]2

M

Þ (b - 1)(b + 1) > 2(2s +1 - 1)2 > 2s(2s + 1 - 1) = (b - 1)(b + 1).

Dẫn đến vô lý.

Hoàn toàn tương tự cho trường hợp 2s – 1(2s + 1 - 1) + 1 = b2. Kết luận n = 7.

Ví dụ 18. Cho a, b nguyên dương nguyên tố cùng nhau, b > 1.

DẠ

Y

Chứng minh rằng với mỗi số nguyên N, tồn tại duy nhất cặp số (x, y) sao cho N = ax + by, 0 ≤ x < b.

Lời giải

a) Tính tồn tại 82


Theo định lý Bezout, luôn tồn tại x, y nguyên sao cho ax + by = N.

L

Chia x cho b ta được x = bq + x0, 0 ≤ x0 < b

FI CI A

Ta được a(bq + x0) + by = n Û ax0 + b(aq + y) = N Û ax0 + by0 = N, y0 = aq + y.

(1)

Tính tồn tại được chứng minh. b) Tính duy nhất.

OF

Từ 0 ≤ x0 < b, ax0 = N – by0 º N (mod b).

Do (a, b) = 1 Þ {0, a, 2a, …, (b - 1)a} là hệ thặng dư đầy đủ mod b.

Do đó tồ tại duy nhất x0 Î {0, 1, …, b - 1} sao cho ax0 º N (mod b). Suy ra số x0 xác định trong (1) là duy nhất.

ƠN

Do đó (x0, y0) duy nhất.

Ví dụ 19. Cho a, b, c là các số nguyên dương đôi một nguyên tố cùng nhau. Chứng minh rằng

NH

N0 = 2abc – ab – bc – ca

là số nguyên lớn nhất không biểu diễn được dưới dạng abx + bcy + caz, với x, y, z là các số nguyên không âm.

QU Y

Lời giải

a) Trước hết ta chứng minh N0 không biểu diễn được dưới dạng abx + bcy + caz. Thật vậy, giả sử tồn tại x, y, z không âm sao cho N0 = abx + bcy + caz Þ 2abc = (x + 1)bc + (y + 1)ca + (z + 1)ab

M

Sử dụng mod a ta suy ra

(x + 1)bc º 0 (mod a) Þ x + 1 º 0 (mod a) Þ x + 1 ≥ a Þ (x + 1)bc ≥ abc.

Tương tự như vậy ta suy ra (x + 1)bc + (y + 1)ca + (z + 1)ab ≥ 3abc > 2abc (vô lý)

Vậy N0 không thể biểu diễn được dưới dạng abx + bcy + caz.

DẠ

Y

b) Tiếp theo ta chứng minh mọi N > N0 thì đều tồn tại x, y, z không âm sao cho N = abx + bcy + caz

Điều đó tương đương với mọi N > 2abc, tồn tại x0, y0, z0 > 0 sao cho

N = abx0 + bcy0 + caz0. 83


Bổ đề: Với a, b là hai số nguyên dương, (a, b) = 1.

L

Khi đó là hệ thặng dư đầy đủ (mod ab). (Bổ đề này rất đơn giản, xin nhường bạn đọc) Trở lại bài toán A = {ax + by| 1 ≤ x ≤ a; 1 ≤ y ≤ b}

OF

là hệ thặng dư đầy đủ (mod ab).

FI CI A

A = {ax + by| 1 ≤ x ≤ a; 1 ≤ y ≤ b}

Mà (c, ab) = 1 nên hệ

B = cA = {cax + bcy| 1 ≤ x ≤ a; 1 ≤ y ≤ b} là hệ thặng dư đầy đủ mod ab.

ƠN

Điều đó có nghĩa là với mọi N nguyên dương, luôn tồn tại y0, z0 sao cho 1≤ y0 ≤ a, 1 ≤ z0≤ b và

N º bcy0 + caz0 (mod ab) Þ N – bcy0 – caz0 = abx0

NH

Þ N =abx0 + bcy0 + caz0

Hơn nữa

N > 2abc Þ abx0 + bcy0 + caz0 > 2abc Þ abx0 + bc(y0 - a) + ca(z0 - b) > 0.

QU Y

Mà y0 – a ≤ 0; z0 – b ≤ 0 nên x0 > 0. Từ đó có điều phải chứng minh.

Nhận xét: Bài toán trên đây gần như được lặp lại ở đề thi TST Ví dụ 20 (Việt Nam TST 2000). Cho a, b, c là ba số nguyên dương đôi một nguyên tố cùng nhau. Số n gọi là “số bướng bỉnh” nếu n không thể biểu diễn được dưới dạng

M

n = abx + bcy + caz với x, y, z là các số nguyên dương. Hỏi có tất cả bao nhiêu số bướng bỉnh?

Lời giải

Gọi A là tập hợp các số bướng bỉnh. a) Theo bài IMO 1983, ta có mọi n > 2abc đều không là số bướng bỉnh.

Y

b) Nếu n = abx + bcy + caz, x, y, z ≥ 1 thì n ≥ ab + bc + ca

DẠ

Do đó mọi n nguyên dương và n ≤ ab + bc + ca – 1 đều là số bướng bỉnh. Từ đó suy ra A ⊂ {1, 2,..., ab + bc + ca − 1} ∪ {ab + bc + ca, ab + bc + ca + 1,..., 2abc} 84


Đặt B = {ab + bc + ca, ab + bc + ca + 1,..., 2abc} .

FI CI A

Ta sẽ chứng minh, với mọi n Î B, ta luôn có n Î A Û f(n) ∉ A.

L

Xét f(n) = ab + bc + ca + 2abc – n.

Thật vậy

Với mọi n Î B, theo chứng minh bổ đề bài trên thì tồn tại x0, y0, z0 sao cho 1 ≤ y0 ≤ a, 1 ≤ z0 ≤ b, và

OF

n = abx0 + bcy0 + caz0

Do n ∉ A Þ x0 ≤ 0 Þ f(n) = ab(1 – x0) + bc(1 + a – y0) + ca(1 + b – z0) Î A Hoàn toàn tương tự, nếu n Î A thì f(n) ∉ A

ƠN

Từ đó suy ra trong B chứa đúng một nửa số các phần tử thuộc A. V ậy

| A |= ab + bc + ca − 1 +

|B| 1 = abc + (ab + bc + ca − 1) 2 2

NH

Ví dụ 21 (VMO 2015). Với a, n nguyên dương, xét phương trình

a2x + 6ay + 36z = n

trong đó x, y, z là các số nguyên không âm.

QU Y

a) Tìm tất cả các giá trị của a để với mọi n ≥ 250, phương trình đã cho luôn có nghiệm x, y, z không âm. b) Biết rằng a > 1, (a, 6) = 1. Tìm giá trị lớn nhất của n theo a sao cho phương trình đã cho không có nghiệm x, y, z nguyên không âm. Lời giải

M

Giả sử với mọi n > 250 luôn tồn tại x, y, z nguyên không âm sao cho n = a2x + 6ay + 36z

Khi đó, (6, a) = 1.

Nếu ngược lại, ta chọn n > 250 sao cho n ⋮ d = (a, 6) thì dẫn đến vô lý.

Y

Ta có

a2x + 6ay + 36z = a(ax + 6y) + 36z = am + 36z.

DẠ

Theo định lý Sylvester thì số N0 = ab – a – b + 1 là số nguyên nhỏ nhất sao cho phương trình ax + by = n

có nghiệm nguyên không âm với mọi n ≥ N0. 85


L

Do đó số 6a2 – a2 – 6 và 36a + 36 – a là các số nguyên dương lớn nhất không biểu diễn được. 6a2 – a2 – 6 < 250 và 36a + 36 – a < 250 Suy ra a < 7. Kết hợp điều kiện (a, 6) = 1 suy ra a = 1 hoặc a = 5.

FI CI A

Từ đó ta phải có

+) Nếu a = 1 thì phương trình n = x + 6y + 36z luôn có nghiệm (n, 0, 0). +) Nếu a = 5, ta chứng minh mọi số n ≥ 250 đều biểu diễn được

OF

n = 25x + 30y + 36z = 25x + 6(5y + 6z - 20) + 120.

Theo định lý Sylvester, mọi số tự nhiên không nhỏ hơn 20 đều biểu diễn được dưới dạng

ƠN

5y + 6z

Suy ra mọi số tự nhiên đều biểu diễn được dưới dạng 5y + 6z – 20 = u. Tương tự, mọi số tự nhiên không nhỏ hơn 120 đều biểu diễn được dưới dạng

NH

25x + 6u.

Do đó, với m ≥ 250 thì n – 120 > 120 nên tồn tại x, u sao cho n – 120 = 25x + 6u Þ n = 25x + 6(u + 20) Lại chọn, y, z sao cho u = 5y + 6z – 20 ta được

QU Y

n = 25x + 30y + 36z

Từ đó được điều phải chứng minh.

b) Ta chứng minh cho trường hợp tổng quát: Cho a, b nguyên dương, (a,b) = 1. Khi đó N = a2b + ab2 – a2 – b2 – ab + 1

M

là số nguyên dương nhỏ nhất để phương trình a2x + aby + b2z = n có nghiệm nguyên không âm.

(Phần này tương tự như bài IMO 1983 – xin nhường lại cho bạn đọc) Ví dụ 22 (DMO 2020) Chứng minh rằng với mọi số nguyên dương k, l cho trước luôn tồn tại vô hạn số nguyên dương m sao cho

k.

Y

i) m

DẠ

ii) gcd(Cmk , l ) = 1. Lời giải

Xét dãy số (un) xác định như sau: u n = k + nl a ,trong đó a là một số nguyên dương sao cho pa > k với mọi p là ước nguyên tố của l. 86


Ta sẽ chứng minh m = un luôn thỏa mãn bài toán.

L

Trước hết, dễ thấy un > k.

Cukn =

(k + nl a )(k − 1 + nl a )...(1 + nl a ) k (k − 1)...1

FI CI A

Ta có

Do với mọi p | l thì pa > k nên vp(i) < vp(nla) nê ta dễ dàng chứng minh được k

v p (i + nl ) = v p ( j ), ∀1 ≤ i ≤ k , ∀p | l  v p (C ) =  (v p (i + nl a ) − v p (i)) = 0 k un

i =1

OF

a

Do đó gcd(Cmk , l ) = 1. Từ đó có đpcm.

ƠN

Nhận xét: Bài này tương tự ý tưởng của bài số chọn đội tuyển Trung Quốc năm 2009, bạn đọc có thể tham khảo trên Aops. Ví dụ 23 (Gặp gỡ toán học 2020)

NH

Chứng minh rằng, với mọi số nguyên dương n và số nguyên tố lẻ p , tổng n  p  

Sn =  (−1)k Cnkp k =0

Lời giải

.

QU Y

chia hết cho p

 n −1   p −1   

Xét y ∈ ℂ là một nghiệm khác 1 của phương trình x p = 1 .

1 + y + y 2 + ... + y p−1 = 0 .

M

Khi đó

k

Vì p là số nguyên tố nên dễ thấy y ≠ 1∀ ( k ; p ) = 1 . Ta có nhận xét sau

DẠ

Y

1 + y k + y 2 k + ... + y ( p −1) k =

y pk − 1 = 0∀ k ⋮ p . yk −1

1 + y k + y 2 k + ... + y ( p −1) k = p∀k ⋮ p .

Xét khai triển nhị thức Newton

( x − 1)

n

= x n + ( −1)Cn1 x n −1 + ... + (−1) n −1 Cnn −1 x + (−1) n . 87


FI CI A

(1 − 1)n = 1n + (−1)Cn1 + ... + ( −1) n −1 Cnn −1 + (−1) n  ( y − 1)n = y n + ( −1)Cn1 y n−1 + ... + ( −1) n −1 Cnn −1 y + ( −1) n .  ... n  p −1 n −1 p −1 y − 1 = y n ( p −1) + (−1)Cn1 y ( )( ) + ... + ( −1) n−1 Cnn−1 y p −1 + ( −1) n ( ) 

L

Lần lượt thay 1, y, y 2 ,..., y p −1 vào khai triển trên ta được:

Cộng vế với vế của tất cả các đẳng thức trên và kết hợp với hai nhận xét trên ta thu được: n

n

n  p  

+ ( y − 1) + ... + ( y p −1 − 1) =  (−1)kp Cnkp . p = p. (−1) k Cnkp = p.Sn . n

OF

(1 − 1)

n  p  

k =0

k =0

p −1 Đặt x1 = 1 − 1; x2 = y − 1;...; x p = y − 1 thì do 1, y, y 2 ,..., y p −1 là p nghiệm phân biệt

( x + 1)

p

ƠN

của phương trình x p = 1 nên x1 , x2 ,..., x p là các nghiệm phân biệt của phương trình

= 1.

n 1

NH

Ta phải chứng minh n 2

n p

x + x + ... + x ⋮ p

QU Y

Ta sẽ làm điều này bằng quy nạp.

 n −1   p −1  +1  

(*).

 n −1

Trước hết ta chứng minh (*) đúng với mọi n ≤ p − 1 . Với n ≤ p − 1 thì   +1 =1  p − 1 Ta có:

p −1

p −1 k

x + x + ... + x =  ( y − 1) =  Ckj (−1) k − j y ji

M

k 1

k 2

k p

i

i =0

k

i =0 j =0

k

p −1

j =0

i =0

=  Ckj ( −1) k − j  y ji = ( −1) p ≡ 0 [ p ] . k

Bây giờ giả sử (*) đúng với mọi n ≤ k ( p − 1), k ∈ ℕ* . Ta chứng minh (*) đúng với mọi

k ( p − 1) < n ≤ ( k + 1)( p − 1), k ∈ ℕ* .

DẠ

Y

Thật vậy, đặt

Tn = x1n + x2n + ... + x np

thì Tn ⋮ p với mọi n . Vì x1 , x2 ,..., x p là các nghiệm phân biệt của phương trình 88


p

−1 = 0

Nên

FI CI A

xip + C1p xip −1 + C p2 xip−2 + ... + C pp −1 xi = 0 ∀i = 1;2;..; p .

 xim+ p−1 + C1p xim+ p−2 + C p2 xim+ p −3 + ... + C pp−1 xim = 0 ∀i = 1;2;..; p và với mọi m ∈ ℕ * . Cộng các vế của các biểu thức trên và cho i chạy ta thu được:

OF

Tm+ p −1 + C1pTm+ p −2 + C p2Tm+ p−3 + ... + C pp−1Tm = 0 với mọi m ∈ ℕ * . Trong đẳng thức trên lần lượt cho

ta thu được Tn ⋮ p

ƠN

m = ( k − 1)( p − 1) + 1;( k − 1)( p − 1) + 2;...; k ( p − 1)  n −1   p −1  +1  

với mọi k ( p − 1) < n ≤ ( k + 1)( p − 1), k ∈ ℕ* .

DẠ

Y

M

QU Y

NH

Bài toán hoàn toàn được chứng minh.

89

L

( x + 1)


L

Phần 2. PHẦN KẾT LUẬN

FI CI A

- Trong chuyên đề này, chúng tôi đã đưa ra các dạng toán quan trọng nhất về số nguyên tố, các định lý sơ cấp cổ điển liên quan đến số nguyên tố và ứng dụng. Ở mỗi chương, chúng tôi đã cố gắng đưa ra hệ thống lý thuyết đầy đủ, các ví dụ được lựa chọn mang tính thời sự và được sắp xếp từ dễ đến khó (theo quan điểm cá nhận của nhóm tác giả). Trong mỗi ví dụ, chúng tôi đã cố gắng phân tích, đưa ra ý tưởng dẫn dắt tới lời giải, phù hợp với thực tiễn giảng dạy học sinh chuyên toán trong thời kì hiện nay.

OF

- Chúng tôi cũng đã cố gắng tuyển chọn, hệ thống các ví dụ từ nhiều nguồn tài liệu khác nhau, chủ yếu từ các đề thi học sinh giỏi các nước gần đây nhất, mục đích làm rõ các dạng toán về số nguyên tố và ứng dụng trong các bài toán số học.

DẠ

Y

M

QU Y

NH

ƠN

- Các ý tưởng về phương pháp giải toán đã được chúng tôi tổng hợp, hình thành trong nhiều năm giảng dạy chuyên toán và một số đã được chúng tôi báo cáo ở những lớp bồi dưỡng giáo viên chuyên hang năm. Hy vọng chuyên đề nhỏ này sẽ góp một phần nhỏ vào quá trình bồi dưỡng đội tuyển học sinh giỏi quốc gia của các đồng nghiệp và chúng tôi cũng rất mong nhận được các ý kiến đóng góp của các thầy cô để chuyên đề ngày càng hoàn thiện hơn.

90


TÀI LIỆU THAM KHẢO

L

[1] Đặng Hùng Thắng, Nguyễn Văn Ngọc, Vũ Kim Thuỷ- Bài giảng Số học - NXBGD, 1997.

[3] Phan Huy Khải - Các chuyên đề Số học - NXBGD, 2005.

FI CI A

[2] Nguyễn Vũ Lương, Nguyễn Ngọc Thắng, Nguyễn Lưu Sơn, Phạm Văn Hùng – Các bài giảng Số học - NXB Đại Học Quốc Gia Hà Nội, 2006. [4] Nguyễn Văn Mậu, Trần Nam Dũng, Đặng Hùng Thắng, Đặng Huy Ruận – Các vấn đề NXBGD, 2008. chọn lọc của Số học -

OF

[6] Titu Andreescu, Dorin Andrica, Zuming Feng - 104 Number theory problems from the training of the USA IMO team - NXB Birkhauser, 2006. [7] Jemes - Elementary Number Theory in nine chapters – NXB Cambridge.

ƠN

[8] Đàm Văn Nhỉ, Phan Đức Hiệp, Lưu Bá Thắng, Trần Thị Hồng Dung, Trần Trung Tình, Văn Đức Chín – Lý thuyết số và chuyên đề nâng cao – NXB Thông tin và truyền thông, 2017.

NH

[9] Đàm Văn Nhỉ, Nguyễn Anh Tuấn, Lê Xuân Dũng, Trần Thị Hồng Dung, Trần Trung Tình, Văn Đức Chín, Đặng Xuân Sơn, Đào Ngọc Dũng – Đa thức – chuỗi và chuyên đề nâng cao – NXB Thông tin và truyền thông, 2017. [10] Lê Anh Vinh, Hoàng Đỗ Kiên, Lê Phúc Lữ, Phạm Đức Hiệp – Định hướng bồi dưỡng học sinh năng khiếu số học – NXB Đại học Quốc gia Hà Nội – 2021.

QU Y

[11] Trần Nam Dũng, Võ Quốc Bá Cẩn, Trần Quang Hùng, Nguyễn Văn Huyện, Lê Phúc Lữ - Các phương pháp giải toán qua các kì thi Olympic (các năm). [12] Nguyễn Duy Liên – Chuyên đề cấp phần tử (Chuyên đề Trại hè Hùng Vương 2014) [13] Phạm Văn Quốc – Bổ đề LTE – Chuyên đề bồi dưỡng giáo viên chuyên. [14] Nguyễn Văn Thảo – Chuyên đề số học.

DẠ

Y

M

[15] Các đề thi trên diễn đàn Aops.

91


Turn static files into dynamic content formats.

Create a flipbook
Issuu converts static files into: digital portfolios, online yearbooks, online catalogs, digital photo albums and more. Sign up and create your flipbook.